Download as pdf or txt
Download as pdf or txt
You are on page 1of 42

SECTION VII

Disorders of the Gastrointestinal System

QUESTIONS

VII-1. All of the following are true about specific functions A. Diarrhea from malabsorption usually improves with
of the gut EXCEPT: fasting, whereas secretory diarrhea persists without
oral intake.
A. Gastric acid sterilizes the upper gut.
B. Sudden awakening from a sound sleep by pain sug-
B. Most nutrient absorption occurs in the small
gests functional rather than organic disease.
intestine.
C. Symptoms from mechanical obstruction, ischemia,
C. The pancreas secretes bicarbonate, which optimizes
inflammatory bowel disease, and functional bowel
the pH in the intestine for enzyme activation.
disorders are alleviated by meals.
D. The stomach secretes intrinsic factor which is neces-
D. Ulcer pain is typically acute in onset.
sary for vitamin B12 absorption.
E. Ulcer symptoms are usually made worse by eating.
E. The terminal ileum is primarily responsible for stool
dehydration, decreasing volumes from 1000–1500 mL VII-5. A 57-year-old female wishes to undergo a screening
to 100–200mL. colonoscopy for colon cancer. She has no family history
of colon cancer and currently has no symptoms referable
VII-2. All of the following diseases are associated with gas-
to the gastrointestinal tract. Which of the following state-
tric acid hypersecretion EXCEPT:
ments is true about colonoscopy as a screening test for
A. Duodenal ulcers colon cancer?
B. G-cell hyperplasia
A. Barium enema is as sensitive as colonoscopy for
C. Pernicious anemia
detecting colitis.
D. Retained antrum syndrome
B. Colonoscopy remains the gold standard for imaging
E. Zollinger-Ellison syndrome
the colonic mucosa.
VII-3. Which of the following gastrointestinal conditions is C. CT colonography has replaced traditional colonos-
characterized by noninflammatory visceral pain? copy for many younger patients because of its ability
to detect serrated polyps with greater sensitivity.
A. Appendicitis D. Flexible sigmoidoscopy would be as effective as colo-
B. Cholecystitis noscopy for detecting colon cancer in this patient.
C. Inflammatory bowel disease E. The cecum can only be reached in 75% of
D. Mesenteric ischemia colonoscopies.
E. Peptic ulcer

VII-4. Which of the following statements regarding gastro-


intestinal symptoms is TRUE?

481
VII-6. Which of the following endoscopic procedures would past 3 years, but none as severe as this event. He denies
warrant antibiotic prophylaxis? food regurgitation outside of these episodes or heartburn
SECTION VII

symptoms. He is able to swallow liquids without difficulty


A. Endoscopic procedures in patients with prosthetic
and has not had any weight loss. Which of the following is
joints
the most likely diagnosis?
B. Endoscopic procedures in patients with synthetic
vascular grafts A. Achalasia
C. Endoscopic ultrasound with fine-needle aspiration B. Adenocarcinoma of the esophagus
in a patient with a lower gastrointestinal tract lesion C. Esophageal diverticula
D. Endoscopic retrograde cholangiopancreatography D. Plummer-Vinson syndrome
Disorders of the Gastrointestinal System

with anticipated complete drainage in a patient with E. Schatzki ring


biliary obstruction in the absence of cholangitis
E. Percutaneous endoscopic feeding tube placement VII-11. A 36-year-old woman with AIDS and a CD4 count
of 35/μL presents with odynophagia and progressive
VII-7. All of the following are indications for urgent endos- dysphagia. The patient reports daily fevers and a 20-lb
copy for acute upper gastrointestinal bleeding EXCEPT: weight loss. The patient has been treated with clotrima-
zole troches without relief. On physical examination, she
A. Bloody nasogastric aspirate that does not clear with
is cachectic with a body mass index of 16 and a weight of
large volume lavage
86 lb. Her temperature is 38.2°C (100.8°F). She is noted to
B. Large volume initial bleed
be orthostatic by blood pressure and pulse. Examination of
C. Orthostatic hypotension
the oropharynx reveals no evidence of thrush. The patient
D. Resting hypotension
undergoes esophagogastroduodenoscopy, which reveals
E. Severe pain
serpiginous ulcers in the distal esophagus without vesicles.
VII-8. Mr. Tyler, a 45-year-old male with known chronic No yellow plaques are noted. Multiple biopsies are taken
alcohol use, presents with two episodes of hematemesis. that show intranuclear and intracytoplasmic inclusions
He recently sustained a fall and injured his knee. He has in large endothelial cells and fibroblasts. What is the best
been taking ibuprofen several times a day for the last week treatment for this patient’s esophagitis?
for pain control. His initial hemoglobin is 10 g/dL. He A. Ganciclovir
receives 2 L of IV fluid and undergoes an urgent upper B. Glucocorticoids
endoscopy. All of the following are true about his risk of C. Fluconazole
rebleeding without intervention EXCEPT: D. Foscarnet
A. A clean-based ulcer is associated with a low risk E. Thalidomide
(3–5%) of rebleeding.
VII-12. A 43-year-old man presents with 6 months of
B. Adherent clots covering the ulcer base have a 90%
worsening dysphagia and postprandial regurgitation. He
risk of rebleeding.
reports difficulty and pain with swallowing both liquids
C. Flat pigmented spots covering the ulcer base have a
and solids; he has no difficulty with the initial components
10% risk of rebleeding.
of swallowing but reports pain in the mid-chest region.
D. When a fibrin plug is seen protruding from a vessel
He frequently regurgitates undigested food 20–60 minutes
wall in the base of an ulcer, (so-called sentinel clot or
after eating or drinking. In the past 2 months, he has lost
visible vessel), the risk of rebleeding from the ulcer is
15 lb. He also has had one episode of presumed pneumonia
40%.
4 months ago notable for a right lower lobe infiltrate. He
E. When active spurting from an ulcer is seen, there is a
has no significant past medical history, takes no medica-
90% risk of ongoing bleeding without therapy.
tions, and does not smoke cigarettes. He works as a service
VII-9. The advantages of endoscopy over barium radiogra-
phy in the evaluation of dysphagia include all of the fol-
lowing EXCEPT:
A. Ability to assess function and morphology
B. Ability to intervene as well as diagnose
C. Ability to obtain biopsy specimens
D. Increased sensitivity for the detection of abnormali-
ties identified by color, e.g., Barrett metaplasia
E. Increased sensitivity for the detection of mucosal
lesions

VII-10. A 47-year-old man is evaluated in the emergency


department for chest pain that developed at a restaurant
after swallowing a piece of steak. He reports intermittent
episodes of meat getting stuck in his lower chest over the
FIGURE VII-12

482
WWW.BOOKBAZ.IR
representative at a major electronics store and has never currently requiring treatment. He undergoes an esophago-
left the United States. Other than signs of recent weight gastroduodenoscopy, which is shown in Figure VII-14A.

SECTION VII
loss, his physical examination is unremarkable. A barium The biopsy is shown in Figure VII-14B.
swallow is performed and shown in Figure VII-12. Which
of the following is the most likely cause of his disease?
A. Autoimmune reaction to latent herpes virus
B. Diffuse spasm on smooth muscle
C. Infection by Trypanosoma cruzi
D. Malignant growth of columnar epithelial cells

QUESTIONS
E. Malignant growth squamous epithelial cells

VII-13. A 64-year-old man with a long history of abdomi-


nal pain, heartburn, and dyspepsia has an esophagogas-
troduodenoscopy to evaluate for peptic ulcer disease. No
gastric or duodenal ulcers are found, but there are tongues
of reddish mucosa extending proximally from the gastroe-
sophageal junction into the esophagus. Biopsies of these
areas demonstrate columnar metaplasia. All of the follow-
ing statements regarding this diagnosis are true EXCEPT:
A. Finding high-grade dysplasia mandates further
intervention.
B. High-dose proton pump inhibitor therapy will likely
cause regression of the mucosal abnormalities.
C. The incidence of these lesions has increased in the
era of potent acid suppression.
D. The patient has a high risk of coexisting cancer.
E. The patient is at significant risk of esophageal
adenocarcinoma.

VII-14. A 35-year-old man presents with progressive dys-


phagia over a 2-month period. He has noted that food has
been getting stuck more frequently in the last 2 weeks. He FIGURE VII-14B
has a history of intermittent gastroesophageal reflux dis-
ease for which he takes over-the-counter antacids every
few weeks. He has a history of eczema as a child but is not Which of the following is the most appropriate first step
in management?
A. Elimination diet
B. IV methylprednisolone
C. Mepolizumab
D. Trial of a proton pump inhibitor
E. Topical corticosteroids

A B

C D
FIGURE VII-14A

483
VII-15. Ms. Pole is a 54-year-old woman who presents with has epigastric pain that is relieved by eating and drinking
recurrent episodes of chest discomfort at rest. She denies milk. He has not had food regurgitation, dysphagia, or
SECTION VII

exertional dyspnea and reports that occasionally her pain bloody emesis or bowel movements. He denies any symp-
is relieved with antacids. She works as a health care con- toms in his chest. Peptic ulcer disease is suspected. Which
sultant and has frequent stressful battles with doctors and of the following statements regarding noninvasive testing
hospitals. An exercise cardiac stress test does not reveal any for Helicobacter pylori is true?
evidence of inducible ischemia. She has a barium swallow,
A. There is no reliable noninvasive method to detect
which is shown in Figure VII-15. You suspect that she has
H. pylori.
diffuse esophageal spasm. Which of the following findings
B. Stool antigen testing is appropriate for proof of cure
Disorders of the Gastrointestinal System

on esophageal manometry would support that diagnosis?


after therapy for H pylori.
C. Plasma antibodies to H. pylori offer the greatest
sensitivity for diagnosis of infection.
D. Exposure to low-dose radiation is a limitation to urea
breath test.
E. False-negative test results using the urea breath test
may occur with recent use of nonsteroidal anti-
inflammatory drugs.

VII-18. A 44-year-old woman complains of 6 months of epi-


gastric pain that is worst between meals. She also reports
symptoms of heartburn. The pain is typically relieved by
over-the-counter antacid medications. She comes to the
clinic after noting her stools darkening. She has no signifi-
cant past medical history and takes no medications. Her
physical examination is normal except for diffuse mid-
epigastric pain. Her stools are heme positive. She under-
goes esophagogastroduodenoscopy, which demonstrates a
well-circumscribed, 2-cm duodenal ulcer that is positive
for Helicobacter pylori. Which of the following is the rec-
FIGURE VII-15 ommended initial therapy given these findings?
A. Lansoprazole, clarithromycin, and metronidazole for
A. Contractions in the distal esophagus with short 14 days
latency relative to the time of the pharyngeal B. Pantoprazole and amoxicillin for 21 days
contraction C. Pantoprazole and clarithromycin for 14 days
B. Hypertensive peristalsis D. Omeprazole, bismuth, tetracycline, and metronida-
C. Impaired lower esophageal sphincter (LES) contrac- zole for 14 days
tion and absent peristalsis with substantial fluid pres- E. Omeprazole, metronidazole, and clarithromycin for
surization of the esophageal body 7 days
D. Impaired LES contraction and absent peristalsis with
no fluid pressurization of the esophageal body VII-19. A 57-year-old man with peptic ulcer disease expe-
E. Impaired LES contraction and absent peristalsis with riences transient improvement with Helicobacter pylori
spastic esophageal contractions eradication. However, 3 months later, symptoms recur
despite acid-suppressing therapy. He does not take non-
VII-16. Which of the following statements regarding ther- steroidal anti-inflammatory drugs. Stool analysis for
apy for gastroesophageal reflux (GERD) therapy is TRUE? H. pylori antigen is negative. Upper gastrointestinal (GI)
A. Clostridium difficile infection rates are decreased in endoscopy reveals prominent gastric folds together with
patients receiving proton pump inhibitors. the persistent ulceration in the duodenal bulb previously
B. Histamine-2 receptor antagonists are more effective detected and the beginning of a new ulceration 4 cm prox-
than proton pump inhibitors. imal to the initial ulcer. Fasting gastrin levels are elevated,
C. The effectiveness of proton pump inhibitors increases and basal acid secretion is 15 mEq/h. What is the best test
substantially with increased dose. to perform to make the diagnosis?
D. Proton pump inhibitors are a rare cause of interstitial A. No additional testing is necessary.
nephritis. B. Blood sampling for gastrin levels following a meal.
E. Weight loss has been proven in randomized con- C. Blood sampling for gastrin levels following secretin
trolled trials to ameliorate the symptoms of GERD. administration.
D. Endoscopic ultrasonography of the pancreas.
VII-17. A 58-year-old man is evaluated for abdominal pain
E. Genetic testing for mutations in the MEN1 gene.
by his primary care physician. He reports severe stress at
the job for the last 3 months and has since noted that he

484
WWW.BOOKBAZ.IR
VII-20. Which of the following are side effects of therapies has lost about 10 lb over the last year. She is otherwise
directed at peptic ulcer disease? healthy and takes no medications. Which of the following

SECTION VII
is the most appropriate recommendation at this point?
A. Famotidine may have weak anti-androgenic side
effects resulting in reversible gynecomastia and A. Increased dietary fiber intake
impotence, primarily in patients receiving high doses B. Measurement of antiendomysial antibody
for prolonged periods of time (months to years). C. Measurement of 24-hour fecal fat
B. Long-term acid suppression, particularly with proton D. Referral to gastroenterologist for endoscopy
pump inhibitors, has been associated with a higher E. Trial of lactose-free diet
incidence of community-acquired pneumonia as well

QUESTIONS
as community- and hospital-acquired Clostridium VII-23. A 54-year-old man is evaluated by a gastroenterolo-
difficile–associated disease. gist for diarrhea that has been present for approximately
C. Magnesium hydroxide can produce constipation and 1 month. He reports stools that float and are difficult to
phosphate depletion. flush down the toilet; these can occur at any time of day
D. Rebound gastric acid hypersecretion has been or night but seem worsened by fatty meals. In addition, he
described in H. pylori–negative individuals after reports pain in many joints lasting days to weeks and not
discontinuation of H2 blockers. It occurs even after relieved by ibuprofen. His wife notes that the patient has
relatively short-term usage (2 months) and may last had difficulty with memory for the last few months. He
for up to 2 months after the medication has been has lost 30 lb and reports intermittent low-grade fevers. He
discontinued. takes no medications and is otherwise healthy. Endoscopy
E. The long-term use of aluminum hydroxide can lead is recommended. Which of the following is the most likely
to milk-alkali syndrome (hypercalcemia, hyperphos- finding on small bowel biopsy?
phatemia with possible renal calcinosis and progres- A. Dilated lymphatics
sion to renal insufficiency). B. Flat villi with crypt hyperplasia
C. Mononuclear cell infiltrate in the lamina propria
VII-21. Which of the following statements regarding gastric
D. Normal small bowel biopsy
acid secretion is true?
E. Periodic acid–Schiff-positive macrophages contain-
A. Basal acid production occurs in a circadian pattern, ing small bacilli
with highest levels occurring during the morning
and lowest levels during the night. VII-24. A 54-year-old man presents with 1 month of diar-
B. Distention of the stomach wall directly activates acid rhea. He states that he has 8 to 10 loose bowel movements
secretion by parietal cells. a day. He has lost 8 lb during this time. Vital signs and
C. Sight, smell, and taste of food are the components of physical examination are normal. Serum laboratory stud-
the cephalic phase, which stimulates gastric secretion ies are normal. A 24-hour stool collection reveals 500 g
via the phrenic nerve. of stool with a measured stool osmolality of 200 mOsm/L
D. The gastric phase is activated once food enters the and a calculated stool osmolality of 210 mOsm/L. Based
stomach. This component of secretion is driven by on these findings, which of the following is the most likely
nutrients (amino acids and amines) that directly (via cause of this patient’s diarrhea?
peptone and amino acid receptors) and indirectly A. Celiac sprue
(via stimulation of intramural gastrin releasing pep- B. Chronic pancreatitis
tide neurons) stimulate the G cell to release gastrin, C. Lactase deficiency
which in turn activates the parietal cell via direct and D. Vasoactive intestinal peptide tumor
indirect mechanisms. E. Whipple disease
VII-22. A 23-year-old woman is evaluated by her primary VII-25. All of the following are direct functions of the intes-
care physician for diffuse, crampy abdominal pain. She tinal epithelium EXCEPT:
reports that she has had abdominal pain for the last sev-
eral years, but it is getting worse and is now associated with A. Barrier and immune defense
intermittent diarrhea without flatulence. This does not B. Fluid and electrolyte absorption and secretion
wake her at night. Stools do not float and are not hard to C. Production of several bioactive amines and peptides
flush. She has not noted any worsening with specific foods, D. Secretion of bile acids
but she does have occasional rashes on her lower legs. She E. Synthesis and secretion of apolipoproteins

485
VII-26. A 40-year-old male with multiple prior admissions A. Children who receive frequent antibiotics in child-
for alcoholic pancreatitis presents for evaluation of several hood are at lower risk of developing IBD.
SECTION VII

months of diarrhea. A 24-hour stool analysis confirms that B. IBD appears with decreasing incidence in countries
he has steatorrhea. Which of the following mechanisms that are becoming more westernized.
likely explains the elevation in stool fat content and result- C. Patients who have had an appendectomy with con-
ing diarrhea? firmed appendicitis have a lower risk of developing
ulcerative colitis.
A. Abnormalities of intestinal lymphatics (e.g., intesti-
D. Peak incidence of ulcerative colitis and Crohn dis-
nal lymphangiectasia)
ease is in the 6th–7th decades of life.
B. Decreased enterohepatic circulation of bile acids
Disorders of the Gastrointestinal System

E. The highest incidence rates of ulcerative colitis and


C. Decreased hydrolysis of triglycerides to free fatty
Crohn disease have been reported in Asia, the Mid-
acids, monoglycerides, and glycerol
dle East, and Northern Africa.
D. Impaired lipid absorption as a result of mucosal
inflammation VII-30. A 36-year-old male has had Crohn disease for 5 years.
E. Increased dietary intake of medium chain triglycerides His disease was initially managed with occasional tapers
of oral prednisone, followed by use of oral budesonide.
VII-27. A 24-year-old woman is admitted to the hospital
Over the last 6 months he has required two courses of oral
with a 1-year history of severe abdominal pain and chronic
prednisone and was hospitalized for worsening abdominal
diarrhea that has been bloody for the past 2 months. She
pain and fevers requiring IV antibiotics and corticoster-
reports a 20-lb weight loss, frequent fevers, and night
oids. He has not had evidence of fistulizing disease. How
sweats. She denies vomiting. Her abdominal pain is
should his Crohn disease be managed at this time?
crampy and primarily involves her right lower quadrant.
She is otherwise healthy. Examination is concerning for an A. Begin IV cyclosporine A
acute abdomen with rebound and guarding present. CT B. Provide intermittent infusions of infliximab each
shows free air in the peritoneum. She is urgently taken to time he has a disease flare
the operating room for surgical exploration, where she is C. Start azathioprine orally at 2–3mg/kg with the poten-
found to have multiple strictures and a perforation of her tial addition of infliximab
bowel in the terminal ileum. The rectum was spared, and a D. Start maintenance prednisone at 0.5mg/kg for 3
fissure from the duodenum to the jejunum was found. The months
perforated area is resected and adhesions lysed. Which of E. Start monthly infusions of IV golimumab
the following findings on pathology of her resected area
confirms her diagnosis? VII-31. Which of the following patients requires no further
testing before making the diagnosis of irritable bowel syn-
A. Crypt abscesses drome and initiating treatment?
B. Flat villi
C. Noncaseating granulomas throughout the bowel wall A. A 76-year-old woman with 6 months of intermittent
D. Special stain for Clostridium difficile toxin crampy abdominal pain that is worse with stress and
E. Transmural acute and chronic inflammation associated with bloating and diarrhea.
B. A 25-year-old woman with 6 months of abdominal
VII-28. A 45-year-old man with ulcerative colitis has been pain, bloating, and diarrhea that has worsened stead-
treated for the last 5 years with infliximab with excellent ily and who now wakes from sleep at night to move
resolution of his bowel symptoms and endoscopic evi- her bowels.
dence of normal colonic mucosa. He is otherwise healthy. C. A 30-year-old man with 6 months of lower abdomi-
He is evaluated by a dermatologist for a lesion that initially nal crampy pain relieved with bowel movements,
was a pustule over his right lower extremity but has since usually loose. Symptoms are worse during the day-
progressed in size with ulceration. The ulcer is moderately time at work and better on the weekend. Weight loss
painful. He does not recall any trauma to the area. On is not present.
examination, the ulcer measures 15 × 7 cm, and central D. A 19-year-old female college student with 2 months
necrosis is present. The edges of the ulcer are violaceous. of diarrhea and worsening abdominal pain with
No other lesions are identified. Which of the following is occasional blood in her stool.
the most likely diagnosis? E. A 27-year-old woman with 6 months of intermittent
abdominal pain, bloating, and diarrhea without asso-
A. Erythema nodosum
ciated weight loss. Crampy pain and diarrhea persist
B. Metastatic Crohn disease
after a 48-hour fast.
C. Psoriasis
D. Pyoderma gangrenosum VII-32. A 29-year-old woman comes to see you in the clinic
E. Pyoderma vegetans because of abdominal discomfort. She feels abdominal
discomfort on most days of the week, and the pain var-
VII-29. Which of the following statements regarding the epi-
ies in location and intensity. She notes constipation as
demiology of inflammatory bowel disease (IBD) is true?

486
WWW.BOOKBAZ.IR
well as diarrhea, but diarrhea predominates. Compared A. Air-fluid levels are commonly seen on plain abdomi-
with 6 months ago, she has more bloating and flatulence nal films.

SECTION VII
than she has had before. She identifies eating and stress as B. Less than 25% of patients present with peritoneal
aggravating factors, and her pain is relieved by defecation. signs.
You suspect irritable bowel syndrome. Laboratory data C. Lower gastrointestinal bleeding will likely be visual-
include white blood cell count 8000/μL, hematocrit 32%, ized on CT angiography.
platelets 210,000/μL, and erythrocyte sedimentation rate D. Thickened colonic wall is not required on CT for the
44 mm/h. Stool studies show the presence of lactoferrin diagnosis of her likely condition.
but no blood. Which of the following interventions is most E. Ultrasound of the pelvis is the best modality to visu-

QUESTIONS
appropriate at this time? alize the likely pathologic process.
A. Antidepressants VII-36. A 67-year-old man is evaluated by the emergency
B. Ciprofloxacin department for blood in the toilet bowl after moving his
C. Colonoscopy bowels. Blood was also present on the toilet paper after
D. Reassurance and patient counseling wiping. He does report straining and recent constipation.
E. Stool bulking agents He has a history of systemic hypertension and hyperlipi-
demia. Vital signs are normal, and he is not orthostatic.
VII-33. Which of the following statements regarding the
Anoscopy shows external hemorrhoids, hematocrit is nor-
pathophysiology of irritable bowel syndrome (IBS) is true?
mal, and bleeding does not recur during his 6-hour emer-
A. Abdominal distention, belching, and flatulence are gency department stay. Which of the following is the most
most commonly due to increased amounts of gas in appropriate management?
the gastrointestinal tract of patients with IBS.
A. Ciprofloxacin and metronidazole
B. Compared with patients without IBS, IBS patients
B. Cortisone suppositories and fiber supplementation
frequently exhibit blunted sensory responses to vis-
C. Hemorrhoidal banding
ceral stimulation.
D. Operative hemorrhoidectomy
C. Gut dysbiosis acting in concert with genetic and
E. Upper endoscopy
environmental factors may alter intestinal permea-
bility, increase antigen presentation resulting in mast VII-37. An 88-year-old woman is brought to your clinic by
cell activation. her family because she has become increasingly socially
D. Almost all patients with IBS display persistent signs withdrawn. The patient lives alone and has been reluc-
of low-grade mucosal inflammation with activated tant to visit or be visited by her family. Family members,
lymphocytes, mast cells, and enhanced expression of including seven children, also note a foul odor in her
proinflammatory cytokines. apartment and on her person. She has not had any weight
E. The majority of IBS patients report a bout of gastro- loss. Alone in the examining room, she only complains
enteritis at the onset of their symptoms. of hemorrhoids. On mental status examination, she does
have signs of depression. Which of the following interven-
VII-34. A 34-year-old female with known irritable bowel
tions is most appropriate at this time?
syndrome has severe bouts of postprandial pain that have
prevented her from being able to enjoy regular meals with A. Head CT scan
her family. Which of the following therapies would be B. Treatment with an antidepressant medication
most likely to improve her discomfort? C. Physical examination including genitourinary and
rectal examination
A. Activated charcoal taken immediately after eating a
D. Screening for occult malignancy
meal.
E. Serum thyroid-stimulating hormone
B. An anticholinergic drug, such as dicyclomine,
administered 30 minutes prior to eating. VII-38. A 59-year-old female develops acute diverticulitis
C. Citalopram administered once daily. and requires hospitalization for IV antibiotics. She has a
D. Fiber supplementation with psyllium. CT scan that does not show evidence of abscess perfora-
E. Small doses of loperamide prior to meals. tion, stricture, or fistula, and she improves in about a week.
Which of the following should be undertaken to prevent
VII-35. A 78-year-old woman is admitted to the hospital with
further symptomatic disease?
fever, loss of appetite, and left lower quadrant pain. She
is not constipated but has not moved her bowels recently. A. Annual colonoscopy
Laboratory examination is notable for an elevated white B. Avoidance of nuts
blood cell count. These symptoms began approximately 3 C. Daily mesalamine therapy
days ago and have steadily worsened. Which of the follow- D. Initiation of a high-fiber diet
ing statements regarding her likely condition is true? E. Subtotal colectomy to reduce the risk of perforation
during a subsequent bout

487
VII-39. A 63-year-old man with poorly controlled type 2 abdominal discomfort and distension. She received a dose
diabetes presents to his primary care physician with 3 days of cefazolin prior to surgery but no other antibiotics. On
SECTION VII

or perianal pain, fever, and difficulty voiding. He has a his- physical examination, she is afebrile with a blood pressure
tory of uncomplicated stage 1 hemorrhoids, which have of 140/80, heart rate of 110 beats/min, respiratory rate of
been treated in the past with cortisone suppositories and a 16 breaths/min, and oxygen saturation of 100% on 2 L of
high-fiber diet. On examination, he is febrile and there is nasal oxygen. She has a distended tympanic abdomen with
a 2-cm fluctuant mass at the anal verge. Which of the fol- absent bowel sounds. There is no rebound tenderness. Her
lowing would be the most appropriate management? upright abdominal film is shown in Figure VII-42. Which
of the following is the most likely diagnosis?
A. Excision and drainage of the mass
Disorders of the Gastrointestinal System

B. Infrared anticoagulation
C. Oral antibiotics and follow-up in 2 days
D. Rubber band ligation
E. Sclerotherapy

VII-40. A 63-year-old man with a history of diabetes and


myocardial infarction was admitted to the medical inten-
sive care unit 1 day ago with sepsis due to pneumococcal
pneumonia with bacteremia. He was started on antibiot-
ics immediately, but he initially required high doses of
noradrenaline and fluids to stabilize his blood pressure.
The noradrenaline was weaned off approximately 12 hours
ago. Over the past 2 hours, he has had increasing abdominal
pain, distension, and bloody stools. His physical examina-
tion is notable for blood pressure of 100/50, regular heart
rate of 100 beats/min, respiratory rate of 22 breaths/min,
and oxygen saturation of 93% on high-flow nasal oxygen.
He has a diffusely tender abdomen with no audible bowel
sounds. An abdominal radiograph shows multiple small
bowel air-fluid levels. Which of the following is the most
likely diagnosis?
A. Arterial embolism
B. Clostridium difficile colitis FIGURE VII-42 Reproduced with permission from Bongard
C. Inflammatory bowel disease FS et al: Current Critical Care Diagnosis & Treatment. New York:
D. Nonocclusive mesenteric ischemia McGraw Hill, 1994.
E. Venous thrombosis

VII-41. A 79-year-old female with known coronary artery A. Acalculous cholecystitis


disease requiring previous percutaneous coronary inter- B. Colonic pseudo-obstruction
vention with stent placement presents to her primary care C. Perforated duodenal ulcer
physician’s office with 3 months of cramping abdominal D. Small bowel obstruction
pain after meals. She denies any change in the pain with E. Small bowel ileus
defecation, and does not report increased flatulence or
belching. She has had more frequent, loose stools but VII-43. A 52-year-old man presents to the emergency
denies constipation. She has lost 15 lb during this time department with acute-onset abdominal pain that awoke
period. Which of the following would be the most appro- him from sleep. He has a history of paroxysmal atrial
priate next step in management? fibrillation but has not had any recent episodes of atrial
fibrillation on remote monitoring. He has no prior history
A. Noncontrast CT scan of the abdomen and pelvis of abdominal surgeries. His pain is 7/10 in severity, and
B. Electrocardiogram to look for the presence of atrial comes in intense waves. He has vomited twice and each
fibrillation time his pain was improved for a few minutes afterward,
C. Initiation of a high fiber diet but then it returned. He is afebrile, with a heart rate in the
D. Initiation of an selective serotonin reuptake inhibitor 120s that is confirmed to be sinus tachycardia by electro-
medication for possible depression cardiogram. He is afebrile. His abdomen is tender and
E. Mesenteric angiography distended. He has high-pitched “musical” bowel sounds.
CT of the abdomen is shown in Figure III-43. He has no
VII-42. A 74-year-old woman is 2 days post hip surgery for
evidence of gallstones or ductal dilatation on right upper
a fracture after a fall. Her only medication prior to admis-
quadrant ultrasound, and the pancreas appears normal on
sion was a calcium supplement, and she has no prior surgi-
cal history. Over the past 24 hours, she has had increasing

488
WWW.BOOKBAZ.IR
VII-46. A 38-year-old man is seen in the urgent care center
with several hours of severe abdominal pain. His symptoms

SECTION VII
began suddenly, but he reports several months of pain in
the epigastrium after eating, with a resultant 10-lb weight
loss. He takes no medications other than over-the-counter
antacids and has no other medical problems or habits. On
physical examination, temperature is 38.0°C (100.4°F),
pulse is 130 beats/min, respiratory rate is 24 breaths/min,
and blood pressure is 110/50. His abdomen has absent

QUESTIONS
bowel sounds and is rigid with involuntary guarding dif-
fusely. A plain film of the abdomen is obtained and shows
free air under the diaphragm. Which of the following is
most likely to be found in the operating room?
A. Necrotic bowel
FIGURE VII-43
B. Necrotic pancreas
CT (not shown). Labs are pending. Which of the following C. Perforated duodenal ulcer
is the most likely cause of his symptoms? D. Perforated gallbladder
E. Perforated gastric ulcer
A. Acute mesenteric ischemia
B. Acute cholecystitis VII-47. Which of the following statements regarding the
C. Acute pancreatitis signs and symptoms of a patient with acute appendicitis
D. Small bowel obstruction is true?
E. Viral gastroenteritis
A. Anorexia is an uncommon symptom.
VII-44. A 32-year-old woman is evaluated in the emergency B. McBurney point describes pain in the mid-
department for abdominal pain. She reports a vague loss epigastrium.
of appetite for the last day and has had progressively severe C. Patients with pelvic appendicitis commonly pre-
abdominal pain, initially at her umbilicus, but now local- sent with dysuria, urinary frequency, diarrhea, or
ized to her right lower quadrant. The pain is crampy. She tenesmus.
has not moved her bowels or vomited. She reports that she D. Right lower quadrant tenderness is present in only
is otherwise healthy and has had no sick contact. Examina- 50% of patients.
tion is notable for a temperature of 38.2°C (100.7°F) and E. Rovsing sign is the most sensitive finding on physical
heart rate of 105 beats/min; otherwise, vital signs are nor- examination.
mal. Her abdomen is tender in the right lower quadrant,
and pelvic examination is normal. Urine pregnancy test VII-48. The greatest source of nutrients and calories in a
is negative. Which of the following imaging modalities is typical individual’s diet come from which source?
most likely to confirm her diagnosis? A. Alcohol
A. CT of the abdomen without contrast B. Carbohydrates
B. Colonoscopy C. Fat
C. Pelvic ultrasound D. Protein
D. Plain film of the abdomen
VII-49. Which of the following factors accounts for the larg-
E. Ultrasound of the abdomen
est amount of water loss per day?
VII-45. The patient in question VII-44 is diagnosed with A. Evaporation and exhalation
acute appendicitis. Which of the following is the most B. Fever
important next step in management? C. Normal stool output (i.e., no diarrhea)
A. Colonoscopy D. Pregnancy
B. Empiric antibiotics and watchful waiting for 48 hours E. Urine output in a person with a very low solute diet
C. Exploratory laparotomy
D. IV corticosteroids
E. Laparoscopic appendectomy

489
VII-50. Mr. Ryan is a 56-year-old man who was admitted VII-52. A 51-year-old alcoholic man presents to the emer-
to the surgical service for care due to exposure injury and gency department complaining of vomiting blood. On
SECTION VII

frostbite in his distal extremities. He has a long-standing further evaluation including gastric lavage, you deter-
history of alcoholism, drinking about 1 L of vodka on a mine that he is not experiencing an upper gastrointestinal
daily basis. You are asked to consult due to concerns of bleed, but he is having significant gingival bleeding. He is
bizarre behaviors exhibited by the patient. He is express- intoxicated and complains of fatigue. Reviewing his chart,
ing a belief that his wounds were the result of burns that you find that he had a hemarthrosis evacuated 6 months
were inflicted on him by “torturers” in the government ago and has been lost to follow-up since then. He takes
because he “knows too much” about government surveil- no medications. Laboratory data show platelets of 250,000
Disorders of the Gastrointestinal System

lance plans. The surgical team reports it is difficult to keep and international normalized ratio of 0.9. He has a diffuse
the patient in his bed, and he seems unsteady on his feet hemorrhagic eruption on his legs that is centered around
at times. He has been medicated throughout his stay for hair follicles. Which of the following is the recommended
prevention of alcohol withdrawal via a symptom-triggered treatment for this patient’s underlying disorder?
approach and last received lorazepam 2 mg orally about
A. Folate
2 hours ago. At that time, the patient was noted to be
B. Niacin
tremulous, tachycardic, and hypertensive. The delusional
C. Thiamine
thoughts are not responsive to treatment of alcohol with-
D. Vitamin C
drawal symptoms. When you see the patient, he is sleeping
E. Vitamin K
quietly. Vital signs are blood pressure 110/82, heart rate
94 beats/min, respiratory rate 16 breaths/min, tempera- VII-53. A 46-year-old male from Mexico was found to be
ture 37.1°C (98.7°F), and SaO2 is 97% on room air. He purified protein derivative–positive on routine health
awakens easily and has a minimal resting tremor. On neu- screening prior to beginning a new job in a hospital. He
rologic examination, he exhibits past-pointing, difficulty was started on isoniazid for treatment of latent tubercu-
with rapid alternating movements, horizontal nystagmus, losis. Three months into his treatment course he develops
and decreased sensation to light touch and pinprick in numbness and tingling in his bilateral feet. On laboratory
the lower extremities below the mid-tibia. His gait is wide assessment his liver function tests are normal. He reports
based and ataxic. He no longer expresses his prior delu- taking his isoniazid faithfully but does not remember
sional beliefs, but he is disoriented and thinks he is in jail. being told to take any additional medications along with
He states he was brought to “this gulag” so that the gov- the isoniazid. Which of the following is the likely cause of
ernment could experiment on him. He has 5% dextrose in his symptoms?
half-normal saline infusing at 100 mL/h and is also receiv-
ing nafcillin 2 g IV every 4 hours for cellulitis. Which of A. Niacin deficiency
the following is the most likely cause of Mr. Ryan’s altered B. Thiamine deficiency
mental state and neurologic findings? C. Vitamin A toxicity
D. Vitamin B6 deficiency
A. Hypoglycemia E. Vitamin B12 deficiency
B. Hyponatremia
C. Niacin deficiency VII-54. A 3-year-old who recently arrived from Indonesia
D. Thiamine deficiency comes in for a routine wellness check with his new adop-
E. Undertreated alcohol withdrawal tive parents. On physical examination he is noted to have
white patches of keratinized epithelium appearing on the
VII-51. A 48-year-old man is diagnosed with carcinoid sclera. Which of the following would be the most appro-
syndrome after presenting with diarrhea, flushing, and priate treatment for this condition?
hypotension. With appropriate treatment, he experiences
an appropriate response biochemically, and his flushing A. Biotin 10 mg/day for 7 days
and blood pressure are markedly improved. However, he B. Pyridoxine 50 mg/day for 14 days
continues to have some mild diarrhea and also has mouth C. Vitamin A 200,000 IU orally on day 1, day 2, and
soreness. He remains fatigued with a loss of appetite and day 15
irritability. On examination, you notice his tongue is D. Vitamin B12 1000 mcg per day for 1 month
bright red and somewhat enlarged. It is tender to touch. E. Vitamin C 100 mg/day for 7 days
In addition, he has a pigmented and scaling rash that is
VII-55. A 21-year-old woman is admitted to the cardiac care
most prominent around his neckline. Which of the follow-
unit after collapsing in her college dormitory. When emer-
ing is the most likely vitamin or mineral deficiency in this
gency personnel arrived, she was found to be in a torsades
patient?
de pointes arrhythmia and was pulseless. She received
A. Copper cardiopulmonary resuscitation, defibrillation, and mag-
B. Niacin nesium en route to the hospital. On arrival, her initial
C. Riboflavin potassium is 1.2 mEq/L. Her physical examination is
D. Vitamin C remarkable for an excessively thin appearance with lanugo
E. Zinc hair on arms and chest. Her body mass index is 14.6 kg/m2.

490
WWW.BOOKBAZ.IR
Which of the following statements is true regarding this VII-60. All of the following are CAGE questions, which
patient’s nutritional state? should be a component of the medical history focusing on

SECTION VII
alcohol abuse and dependence, EXCEPT:
A. Mortality in the disease is most commonly due to
complications of malnutrition. A. Do you feel like you have a greater tolerance for alco-
B. Poor wound healing and frequent skin infections are hol than your friends?
common complications. B. Have you ever felt you ought to cut down on your
C. Systemic inflammation is a predominant finding on drinking?
laboratory examination. C. Have people annoyed you by criticizing your
D. The serum albumin is typically less than 2.8 g/dL. drinking?

QUESTIONS
E. Triceps skinfold <3 mm and mid-arm muscle cir- D. Have you ever felt guilty or bad about your drinking?
cumference <15 cm are useful diagnostic criteria. E. Have you ever had a drink first thing in the morning
to steady your nerves or get rid of a hangover?
VII-56. You are caring for a 54-year-old woman in the inten-
sive care unit who was admitted for treatment of severe VII-61. A 43-year-old female presents to her primary care
sepsis and pneumonia. You would like to initiate enteral physician with fatigue and malaise. On routine laboratory
nutrition and plan to calculate basal energy expenditure testing she is found to have an aspartate aminotransferase
for the patient. All of the following factors are used to of 150 IU/L, and alanine aminotransferase of 165 IU/L,
determine the patient’s caloric needs EXCEPT: an alkaline phosphatase of 125 IU/L, a total bilirubin of
1.2 mg/dL, and an international normalized ratio of 0.9.
A. Age
Which of the following best describes the pattern of liver
B. Albumin
tests?
C. Gender
D. Height A. Cholestatic
E. Weight B. Cirrhotic
C. Hepatocellular
VII-57. Which of the following conditions would be best D. Mixed
classified as starvation-associated malnutrition? E. Peritonitic
A. Closed head injury
VII-62. All of the following are potential advantages of mag-
B. Major depression
netic resonance cholangiopancreatography over endo-
C. Motor-vehicle accident
scopic retrograde cholangiopancreatography EXCEPT:
D. Pancreatic cancer
E. Sarcopenic obesity A. Faster image acquisition
B. Higher sensitivity for identifying ampullary lesions
VII-58. Which of the following statements regarding the C. No risk of ionizing radiation
methodologies for determining body composition and D. No risk of pancreatitis
nutritional status is true? E. No need for contrast media
A. Albumin is useful in assessing nutritional status
VII-63. Elevation in all of the following laboratory studies
because of its long half-life.
would be indicative of liver disease EXCEPT:
B. Anthropomorphic analyses such as measur-
ing skinfolds and circumferences are reliable and A. 5′-Nucleotidase
reproducible. B. Aspartate aminotransferase
C. Bioelectrical impedance can be readily applied and C. Conjugated bilirubin
interpreted in individuals of different races and eth- D. Unconjugated bilirubin
nic groups. E. Urine bilirubin
D. C-reactive protein is a specific marker for
malnutrition. VII-64. Which of the following statements regarding liver
E. Dual energy x-ray absorptiometry scanning can be function tests is true?
used to compare truncal and appendicular soft tissue. A. Alanine aminotransferase (ALT) is found in liver,
cardiac muscle, skeletal muscle, and kidney.
VII-59. Which of the following is the most common symp-
B. Elevation of aspartate aminotransferase (AST) and
tom or sign of liver disease?
ALT to >1000 IU/L is typical of ischemic hepatitis.
A. Fatigue C. Elevation of AST is more specific for liver dysfunc-
B. Itching tion than elevation of ALT.
C. Jaundice D. Increased AST and ALT with an AST:ALT ratio of >3
D. Nausea is typical of acute viral hepatitis.
E. Right upper quadrant pain E. The magnitude of elevated AST and ALT has impor-
tant prognostic significance in acute hepatitis.

491
VII-65. Which of the following is true about aspartate ami- A. Bilirubin values <4 mg/dL imply concomitant gall-
notransferase (AST) and alanine aminotransferase (ALT) bladder or biliary dysfunction.
SECTION VII

in liver injury? B. Bilirubin values >4 mg/dL (68 μmol/L) imply con-
comitant liver dysfunction.
A. A ratio of ALT:AST greater than 1 is suggestive of
C. It is typically composed of 50% conjugated and 50%
cirrhosis.
unconjugated bilirubin.
B. ALT is usually equal to or greater than AST in most
D. Prolonged hemolysis may result in the development
causes of acute liver injury.
of cirrhosis.
C. AST is often normal is cases of alcoholic cirrhosis.
D. AST and ALT usually remain elevated for days after
Disorders of the Gastrointestinal System

VII-69. A 26-year-old female who is 3-months pregnant


the passage of a gallstone. presents for a routine prenatal visit and is found to be
E. If the ALT:AST ratio is greater than 3, then alcoholic jaundiced. This is her first pregnancy and she reports no
hepatitis should be suspected. symptoms other than occasional early morning nausea
that resolves after eating breakfast. She has no pruritus. Her
VII-66. Which of the following is true about biochemical
physical examination is unremarkable with the exception
liver testing?
of obvious jaundice. She has no hepatosplenomegaly and
A. γ-Glutamyl transpeptidase elevation is specific for her fundus is not yet palpable. Labs are notable for a total
cholestatic liver injury. bilirubin of 6 mg/dL, with a direct bilirubin of 5 mg/dL.
B. Increased IgM levels may be a marker of autoim- Her aspartate aminotransferase, alanine aminotransferase,
mune hepatitis. and alkaline phosphatase levels are all within the normal
C. Low factor VIII levels may indicate chronic liver range. Which of the following is the most likely cause of
disease. her hyperbilirubinemia?
D. Patients over the age of 60 can have a mild elevation
A. Benign recurrent intrahepatic cholestasis
in their alkaline phosphatase levels up 1.5 times the
B. Cholestasis of pregnancy
upper limit of normal.
C. Crigler-Najjar syndrome type 2
E. Serum albumin is a good marker of the severity of
D. Dubin-Johnson syndrome
acute liver injury.
E. HELLP (hemolysis, elevated liver enzymes, and low
VII-67. A 26-year-old male resident is noticed by his attend- platelet) syndrome
ing physician to have yellow eyes after his 24-hour call
VII-70. A 34-year-old man presents to the physician com-
period. When asked, the resident states he has no medi-
plaining of yellow eyes. For the past week, he has felt ill,
cal history and is a social drinker, but on occasion, he
with decreased oral intake, low-grade fevers (~100°F),
has thought he might have mild scleral icterus when he
fatigue, nausea, and occasional vomiting. With the onset
is stressed or has more than four to five alcoholic drinks.
of jaundice, he has noticed pain in his right upper quad-
He never sought medical treatment because his eyes would
rant. He currently uses marijuana and ecstasy and has a
return fully to normal within 2 days. He denies nausea,
prior history of injection drug use with cocaine. He has
abdominal pain, dark urine, light-colored stools, pruri-
no other past medical history, but he was unable to donate
tus, or weight loss. On examination, he has a body mass
blood for reasons that he cannot recall 4 years previously.
index of 20.1 kg/m2, and his vital signs are normal. Scleral
His social history is remarkable for working as a veterinary
icterus is present. There are no stigmata of chronic liver
assistant. On sexual history, he reports five male sexual
disease. The patient’s abdomen is soft and nontender. The
partners over the past 6 months. He does not consistently
liver span is 8 cm to percussion. The liver edge is smooth
use condoms. On physical examination, he appears ill and
and palpable only with deep inspiration. The spleen is not
has obvious jaundice with scleral icterus. His liver is 15 cm
palpable. Laboratory examinations are normal except for a
to percussion, palpable 6 cm below the right costal margin.
total bilirubin of 3.0 mg/ dL. Direct bilirubin is 0.2 mg/dL.
The edge is smooth and tender to palpation. The spleen is
Aspartate aminotransferase, alanine aminotransferase,
not enlarged. There are no stigmata of chronic liver dis-
and alkaline phosphatase are normal. Hematocrit, lactate
ease. His aspartate aminotransferase is 1232 IU/L, alanine
dehydrogenase, and haptoglobin are normal. Which of the
aminotransferase is 1560 IU/L, alkaline phosphatase is
following is the most likely diagnosis?
394 IU/L, total bilirubin is 13.4 mg/dL, and direct bilirubin
A. Autoimmune hemolytic anemia is 12.2 mg/dL. His international normalized ratio is 2.3,
B. Crigler-Najjar syndrome type 1 and activated partial thromboplastin time is 52 seconds.
C. Choledocholithiasis Hepatitis serologies are sent and reveal the following:
D. Dubin-Johnson syndrome
Hepatitis A IgM: negative
E. Gilbert syndrome
Hepatitis A IgG: negative
VII-68. Which of the following statements regarding the Hepatitis B core IgM: positive
hyperbilirubinemia seen in patients with significant intra-
Hepatitis B core IgG: negative
vascular hemolysis is true?
Hepatitis B surface antigen: positive

492
WWW.BOOKBAZ.IR
Hepatitis B surface antibody: negative tender liver below the right costal margin. In regard to
acute hepatitis, which of the following is true?

SECTION VII
Hepatitis B e antigen: positive
Hepatitis B e antibody: negative A. A distinction between viral etiologies cannot be
Hepatitis C antibody: positive made using clinical criteria alone.
B. Based on age and risk factors, he is likely to have a
What is the cause of the patient’s current clinical hepatitis B infection.
presentation? C. He does not have hepatitis E virus, as this infects only
A. Acute hepatitis A infection pregnant women.
D. This patient cannot have hepatitis C because his pres-

QUESTIONS
B. Acute hepatitis B infection
C. Acute hepatitis C infection entation is too acute.
D. Chronic hepatitis B infection E. This patient does not have hepatitis A because his
E. Drug-induced hepatitis presentation is too fulminant.

VII-71. In the patient described in question VII-70, what VII-74. Which of the following best explains why neonatal
would be the best approach to prevent development of infection with hepatitis B virus (HBV) does not result in
chronic hepatitis? significant hepatocellular inflammation early in life?

A. Administration of anti-hepatitis A virus IgG A. Effective priming of HBV-specific T cells with HBV
B. Administration of lamivudine leads to the lack of an immune response.
C. Administration of pegylated interferon-α plus B. HBV does not easily cross the placenta.
ribavirin C. In utero exposure to HBV induces immune tolerance.
D. Administration of prednisone beginning at a dose of D. Maternal antibodies against hepatitis core antigen
1 mg/kg daily result in immunity against infection.
E. Do nothing and observe as 99% of individuals with E. Natural killer cells are not particularly effective early
this disease recover in life.

VII-72. A 16-year-old girl had visited your clinic 1 month ago VII-75. A 57-year-old female presents to your clinic for a
with jaundice, vomiting, malaise, and anorexia. Two other routine health evaluation. She is complaining of intermit-
family members were ill with similar symptoms. Based on tent abdominal pain so you decide to order serum chemis-
viral serologies, including a positive anti-hepatitis A virus tries. Her aspartate aminotransferase is mildly elevated at
IgM, a diagnosis of hepatitis A was made. The patient was 75 IU/L, and her alanine aminotransferase is also mildly
treated conservatively, and 1 week after first presenting, elevated at 66 IU/L. Her bilirubin and international nor-
she appeared to have made a full recovery. She returns to malized ratio are normal. She denies recent alcohol use
your clinic today complaining of the same symptoms she and has never used injection drugs. She is sexually active
had 1 month ago. She is jaundiced, and an initial panel and has had two recent male partners. She does not use
of laboratory tests returns elevated transaminases. Which barrier protection. She was born in China but has lived in
of the following offers the best explanation of what has the United States for 40 years. You decide to check hepati-
occurred in this patient? tis serologies and obtain the following results:

A. Co-infection with hepatitis C Hepatitis B surface antigen: positive


B. Inappropriate treatment of initial infection Hepatitis B surface antibody: negative
C. Incorrect initial diagnosis; this patient likely has hep- Hepatitis B core antibody: positive
atitis B Hepatitis B e-antigen: negative
D. Reinfection with hepatitis A Hepatitis B e-antibody: positive
E. Relapse of hepatitis A
Which of the following best describes her hepatitis B
VII-73. An 18-year-old man presents to a rural clinic with status?
nausea, vomiting, anorexia, abdominal discomfort, myal- A. Acute hepatitis B
gias, and jaundice. He describes occasional alcohol use and B. Chronic Hepatitis B, high infectivity
is sexually active. He describes using heroin and cocaine “a C. Immunization against hepatitis B
few times in the past.” He works as a short-order cook in D. Late acute or chronic hepatitis B, low infectivity
a local restaurant. He has lost 15.5 kg (34 lb) since his last E. Recovery from hepatitis B
visit to the clinic and appears emaciated and ill. On exami-
nation, he is noted to have icteric sclerae and a palpable,

493
VII-76. Which of the following statements is true about the A. Acute hepatocellular injury due to INH is an idio-
prevention of viral hepatitis? syncratic reaction that will manifest within the first
SECTION VII

2 months of initiation of therapy.


A. Health care workers in endemic areas should receive
B. Controlled trials have demonstrated that monthly
the hepatitis D vaccine every 5 years.
monitoring of aminotransferase levels reduces mor-
B. Patients with acute hepatitis A should be placed on
bidity in U.S. health care workers receiving INH
contact isolation to prevent transmission to health
prophylaxis.
care workers and other patients.
C. Elevation of aminotransferase levels in the first
C. Serum Ig has been shown to be effective in postexpo-
2 months of therapy is an indication to stop INH and
sure prophylaxis for hepatitis C.
Disorders of the Gastrointestinal System

switch to another drug.


D. The hepatitis B vaccine is a live attenuated vaccine
D. The patient has a 50–70% chance of transient ele-
that should be avoided in immunocompromised
vation of his aminotransferase levels in the first
patients.
2 months of treatment.
E. There is an effective vaccine against hepatitis E
E. The frequency of acute hepatocellular injury due
genotype 1 that also has effectiveness against other
to INH is age dependent, increasing in patients
genotypes.
>35 years of age.
VII-77. A 32-year-old woman is admitted to the inten-
VII-79. Which of the following statements is true about
sive care unit following an overdose of acetaminophen
drug-induced liver injury (DILI)?
with co-ingestion of alcohol. She was known to be alert
and interactive about 4 hours prior to her presentation A. Direct toxic hepatitis is usually clinically indistin-
when she had a fight with her boyfriend who then left guishable from viral hepatitis.
the home. When he returned 6 hours later, he found an B. Direct toxic hepatitis occurs with predictable regu-
empty bottle of acetaminophen 500-mg capsules as well larity after exposure and is dose dependent.
as an empty vodka bottle. The exact number of pills in the C. “Idiosyncratic” drug reactions are easily identified by
bottle is unknown, but the full bottle held as much as 50 their histologic pattern on liver biopsy.
capsules. The patient was unresponsive and had vomited, D. The incidence of DILI is higher in patients with
so her boyfriend called 911. On arrival to the emergency chronic liver disease.
department, the patient is stuporous. Her vital signs are E. The occurrence of jaundice in a phase 3 clinical trial
pulse 109 beats/min, respiratory rate 20 breaths/min, is not predictive of severe hepatotoxicity in post-
blood pressure 96/52, and oxygen saturation 95% on room marketing surveillance.
air. Her examination shows mild nonspecific abdominal
pain with palpation. The liver is not enlarged. Her initial VII-80. Which of the following statements regarding drug-
laboratory values show a normal complete blood count, induced liver injury is true?
normal electrolytes, and kidney function. The aspartate A. Amiodarone-induced liver injury resolves within
aminotransferase is 68 IU/L, alanine aminotransferase is days of stopping the drug.
46 IU/L, alkaline phosphatase is 110 IU/L, and total bili- B. Phenytoin-induced liver injury is only rarely accom-
rubin is 1.2 mg/dL. Glucose and coagulation studies are panied by systemic symptoms and is usually picked
normal. The serum alcohol level is 210 g/dL. The acetami- up on screening liver enzyme testing.
nophen level is 350 μg/mL. What is the most appropriate C. Sodium valproate liver toxicity is more common in
next step in the treatment of this patient? patients with mitochondrial deficiencies and can be
A. Administration of activated charcoal or ameliorated by the administration of IV carnitine.
cholestyramine D. The incidence of amoxicillin-induced liver injury is
B. Administration of N-acetylcysteine 140 mg/kg fol- decreased when administered in combination with
lowed by 70 mg/kg every 4 hours for a total of clavulanate.
15–20 doses E. When it occurs, nitrofurantoin liver toxicity happens
C. Continued monitoring of liver function, glucose, and soon after the initiation of the drug.
coagulation studies every 4 hours with administra-
VII-81. A 38-year-old woman is evaluated for elevated
tion of N-acetylcysteine if these begin to change
transaminase levels that were identified during routine
D. Do nothing as normal liver function tests and coagu-
laboratory testing for life insurance. She is originally from
lation studies are indicative of only a minor ingestion
Thailand and immigrated to the United States 10 years
E. Initiate hemodialysis for toxin clearance
previously. She has been married to an American for the
VII-78. A 31-year-old health care worker is found to have a past 12 years, meeting him while he was living abroad
newly positive tuberculin skin test 6 weeks after an expo- for business. She previously worked in Thailand as a
sure to a patient with active pulmonary tuberculosis. He is deputy tourism minister for the government but is not
asymptomatic and has a normal chest radiograph. Which currently employed. She has no significant past medical
of the following statements regarding initiation of isonia- history. She had one uncomplicated pregnancy at the age
zid (INH) prophylactic therapy is true? of 22. When queried about risk factors for liver disease,
she denies alcohol intake or drug abuse. She has never

494
WWW.BOOKBAZ.IR
had a blood transfusion. She recalls an episode of jaun- upper limit of normal he does not currently require
dice that she did not seek evaluation for about 15 years treatment.

SECTION VII
ago as it resolved spontaneously. She currently feels well,
and her husband wished to have her added to his life VII-84. A 34-year-old woman is evaluated for fatigue,
insurance policy. On physical examination there are no malaise, arthralgias, and a 10-lb weight loss over the past
stigmata of chronic liver disease. Her laboratory studies 6–8 weeks. She has no past medical history. Since feeling
reveal an aspartate aminotransferase of 346 IU/L, alanine poorly, she has taken approximately one or two tablets
aminotransferase of 412 IU/L, alkaline phosphatase of of acetaminophen 500 mg daily. On physical examina-
98 IU/L, and total bilirubin 1.5 of mg/dL. Further workup tion, her temperature is 37.9°F (100.2°F), respiratory rate

QUESTIONS
includes the following viral studies: hepatitis A IgG posi- is 18 breaths/min, blood pressure is 100/48, heart rate is
tive, hepatitis B surface antigen positive, hepatitis B e anti- 92 beats/min, and oxygen saturation is 96% on room air.
gen positive, anti-hepatitis B virus core IgG positive, and She has scleral icterus. Her liver edge is palpable 3 cm
hepatitis C IgG negative. The hepatitis B virus DNA level below the right costal margin. It is smooth and tender.
is 4.8 × 104 IU/mL. Which of the following medications is The spleen is not enlarged. She has mild synovitis in the
indicated for this patient? small joints of her hands. Her aspartate aminotransferase
is 542 IU/L, alanine aminotransferase is 657 IU/L, alkaline
A. Acyclovir phosphatase is 102 IU/L, total bilirubin is 5.3 mg/dL, and
B. Entecavir direct bilirubin is 4.8 mg/dL. Which of the following tests
C. Ritonavir would be LEAST likely to be positive in this diagnosis?
D. Simeprevir
E. No treatment is necessary A. Antinuclear antibodies in a homogeneous pattern
B. Anti-liver/kidney microsomal antibodies
VII-82. A 46-year-old man is known to have chronic hepati- C. Antimitochondrial antibodies
tis C virus (HCV) infection. He is a former IV drug user for D. Hypergammaglobulinemia
more than 20 years who has been abstinent from drug use E. Rheumatoid factor
for 1 year. He was treated for tricuspid valve endocarditis
3 years previously. He does not know when he acquired VII-85. Which of the following statements is true about the
HCV. His laboratory studies show a positive HCV IgG mechanism of liver injury in patients with autoimmune
antibody with a viral load of greater than 1 million copies. hepatitis?
The virus is genotype 2. His aspartate aminotransferase is A. Acute viral hepatitis has not been associated with the
82 IU/L, and his alanine aminotransferase is 74 IU/L. He subsequent development of autoimmune hepatitis.
undergoes liver biopsy, which demonstrates a moderate B. Circulating autoantibodies in patients have been
degree of bridging fibrosis. Which of the following is the directly linked to hepatocyte injury.
most predictive of the development of cirrhosis? C. Immune complex deposition is an important mecha-
A. Abnormal transaminases nism of hepatocyte injury.
B. Bridging fibrosis on liver biopsy D. Macrophages are the main effector cell of liver injury.
C. Genotype 2 E. Molecular mimicry by cross-reacting antigens that
D. History of bacterial endocarditis contain epitopes similar to liver antigens is postu-
E. History of IV drug use lated to activate cytotoxic T cells.

VII-83. For the patient in described in question VII-82, VII-86. All of the following statements regarding alcoholic
which of the following statements is true about potential liver disease are true EXCEPT:
treatment options for his hepatitis C virus? A. Fatty liver is present in >90% of daily and binge
A. A liver biopsy should be performed to determine the drinkers.
appropriate treatment regimen. B. Hepatitis C infection worsens the prognosis of alco-
B. Combination therapy with sofosbuvir and velpatasvir holic liver disease.
would be a reasonable first-line treatment based on C. Over 50% of alcoholics will develop alcoholic
his genotype. hepatitis.
C. He should be treated with ribavirin and pegylated D. Quantity and duration of alcohol consumption are
interferon as first-line agents. the most important risk factors for the development
D. Monotherapy with boceprevir would avoid any of alcoholic liver disease.
potential interactions with the CYP3A4 pathway. E. The pathologic hallmarks of alcoholic liver disease
E. Because his alanine aminotransferase and aspartate are fatty liver, hepatitis, and cirrhosis.
aminotransferase are not more than three times the

495
VII-87. A 32-year-old woman is admitted to the hospi- marked fatty infiltration of the liver. Laboratory studies
tal with fever, abdominal pain, and jaundice. She drinks show his transaminases are 2× normal with normal alka-
SECTION VII

approximately 6 beers daily and has recently increased line phosphatase, bilirubin, and prothrombin time. Other
her alcohol intake to more than 12 beers daily. She has no than insulin, he takes no medications, does not consume
other substance abuse history and has no prior history of alcohol or illicit drugs, and has no family history of liver
alcoholic liver disease or pancreatitis. She is not taking any disease. On physical examination, he is obese (body mass
medications. On physical examination, she appears ill and index 32 kg/m2) with normal vital signs and no other
disheveled with a fruity odor to her breath. Her vital signs abnormalities. You think he likely has nonalcoholic fatty
are heart rate 122 beats/min, blood pressure 95/56, respir- liver disease (NAFLD). All of the following statements
Disorders of the Gastrointestinal System

atory rate 22 breaths/min, temperature 38.4°C (101.2°F), regarding his potential therapy are true EXCEPT:
and oxygen saturation 98% on room air. She has scleral
A. Bariatric surgery is safe in patients with NAFLD.
icterus, and spider angiomata are present on the trunk.
B. Exercise may reduce hepatic steatosis.
The liver edge is palpable 10 cm below the right costal
C. Statins may worsen inflammation in NAFLD.
margin. The liver is smooth and tender to palpation. The
D. There are no therapies approved by the Food and
spleen is not palpable. No ascites or lower extremity edema
Drug Administration for NAFLD.
is present. Laboratory studies demonstrate an aspartate
E. Vitamin E may reduce aminotransferase levels and
aminotransferase of 431 IU/L, alanine aminotransferase of
hepatic steatosis.
198 IU/L, bilirubin of 8.6 mg/dL, alkaline phosphatase of
201 IU/L, amylase of 88 U/L, and lipase of 50 U/L. Total VII-90. Which of the following statements about non-
protein is 6.2 g/dL, and albumin is 2.8 g/dL. The pro- alcoholic steatohepatitis (NASH) is true?
thrombin time is 29 seconds (control, 13 seconds) with
international normalized ratio of 2.2. What is the best A. Insulin resistance is an important driver of lipid
approach to treatment of this patient? uptake, fat synthesis, and fat storage that leads to tri-
glyceride accumulation in hepatocytes.
A. Administer IV fluids, thiamine, and folate and B. Greater than 50% of patients with nonalcoholic fatty
observe for improvement in laboratory tests and liver disease (NAFLD) also have some component of
clinical condition. NASH.
B. Administer IV fluids, thiamine, folate, and imipenem C. Serum biomarkers are useful in distinguishing
while awaiting blood culture results. NAFLD from NASH.
C. Administer prednisone 40 mg daily for 4 weeks D. The contribution of NASH to the development of cir-
before beginning a taper. rhosis is on the decline in the United States.
D. Consult surgery for management of acute E. Triglyceride accumulation within hepatocytes is
cholecystitis. directly cytotoxic.
E. Perform an abdominal CT with IV contrast to assess
for necrotizing pancreatitis. VII-91. A 63-year-old man with cirrhosis and portal hyper-
tension due to hemochromatosis presents with altered
VII-88. Which of the following is true about the pathophysi- mental status. He has chronic ascites controlled with
ology of alcoholic liver disease? diet and spironolactone. He has a history of one esopha-
A. Alcoholic fatty liver disease is a benign condition and geal bleed but none since starting propranolol. His fam-
does not lead to lasting liver injury. ily reports that over the last 2 days, he has become more
B. Alcoholic fatty liver disease is characterized by bal- confused, but he has had no melena or hematemesis. He is
looning degeneration, spotty necrosis, polymorpho- afebrile with normal vital signs, and physical examination
nuclear infiltrate, and fibrosis in the perivenular and is notable for ascites, asterixis, and being oriented only to
perisinusoidal space of Disse. person. His laboratory examination is notable for a hemo-
C. Fatty accumulation begins in the perivenular hepato- globin of 10.1 g/dL (baseline 9.5), creatinine of 1.4 mg/dL
cytes and can eventually involve the entire hepatic (baseline 1.4), and blood urea nitrogen of 45 mg/dL (base-
lobule. line 18). A paracentesis is performed that yields reveals
D. It is relatively easy to differentiate alcoholic from clear fluid with 800 white blood cells/μL (40% neutrophils).
nonalcoholic fatty liver disease on liver biopsy. Which of the following is the most indicated therapy?
E. Once fatty accumulation occurs in the liver, it is not A. Ampicillin, ceftriaxone, vancomycin
reversible. B. Cefotaxime
C. Esophagogastroduodenoscopy with banding
VII-89. A 44-year-old man seeks evaluation for an abnor-
D. Hemodialysis
mal finding on abdominal ultrasonography. He has a his-
E. Lactulose
tory of type 2 diabetes mellitus and is on insulin therapy.
Last week, he was evaluated in the emergency department VII-92. A 48-year-old woman presents complaining of
for mid-epigastric pain likely due to nonsteroidal anti- fatigue and itching. She has been tired for the past
inflammatory drug therapy for muscle aches (he recently 6 months and recently has developed itching diffusely. It
started exercising because his wife told him to lose weight). is worse in the evening hours but is intermittent. She does
During the evaluation, an abdominal ultrasound showed

496
WWW.BOOKBAZ.IR
not note it to be worse following hot baths or showers. Her A. 24-Hour urine copper
past medical history is significant only for hypothyroid- B. Antimitochondrial antibodies

SECTION VII
ism for which she takes levothyroxine 125 μg daily. On C. Endoscopic retrograde cholangiopancreatography
physical examination, she has mild jaundice and scleral D. Hepatitis B serologies
icterus. The liver is enlarged to 15 cm on palpation and is E. Serum ferritin
palpable 5 cm below the right costal margin. Xanthomas
are seen on both elbows. Hyperpigmentation is notice- VII-95. A 55-year-old man with cirrhosis thought second-
able on the trunk and arms where the patient has exco- ary to nonalcoholic steatohepatitis presents with altered
riations. Laboratory studies demonstrate the following: mental status. All of the following can precipitate hepatic

QUESTIONS
white blood cells 8900/μL, hemoglobin 13.3 g/dL, hema- encephalopathy in this type of patient EXCEPT:
tocrit 41.6%, and platelets 160,000/μL. The creatinine is A. Dehydration
1.2 mg/dL. The aspartate aminotransferase is 52 IU/L, ala- B. Hyperkalemia
nine aminotransferase is 62 IU/L, alkaline phosphatase is C. Hypokalemia
216 IU/L, total bilirubin is 3.2 mg/dL, and direct bilirubin D. Medication nonadherence
is 2.9 mg/dL. The total protein is 8.2 g/dL, and albumin is E. Spontaneous bacterial peritonitis
3.9 U/L. The thyroid-stimulating hormone is 4.5 U/mL.
Antimitochondrial antibodies are positive. Perinuclear VII-96. A 64-year-old man with known cirrhosis is admitted
antineutrophil cytoplasmic antibodies (ANCA) and cyto- to the intensive care unit with a large gastrointestinal bleed
plasmic ANCA are negative. What is the most likely cause and altered mental status. He is confused and unable to
of the patient’s symptoms? provide any history. His initial hemoglobin is 6.9 g/dL, and
his vital signs are notable for a heart rate of 115 beats/min
A. Lymphoma
and a blood pressure of 90/55. In addition to fluid resus-
B. Polycythemia vera
citation including transfusion of packed red blood cells,
C. Primary biliary cirrhosis
all of the following are appropriate therapy at this time
D. Primary sclerosis cholangitis
EXCEPT:
E. Uncontrolled hypothyroidism
A. Endoscopic sclerotherapy
VII-93. A 42-year-old man with cirrhosis related to hepatitis B. Endoscopic variceal ligation
C and alcohol abuse has ascites requiring frequent large- C. Octreotide
volume paracentesis. All of the following therapies would D. Propranolol
be indicated for this patient EXCEPT: E. Transjugular intrahepatic portosystemic shunt
A. Fluid restriction to less than 2 L daily
VII-97. In the patient in question VII-96, all of the following
B. Furosemide 40 mg daily
could be used as prophylaxis for further variceal bleeding
C. Sodium restriction to less than 2 g daily
EXCEPT:
D. Spironolactone 100 mg daily
E. Transjugular intrahepatic portosystemic shunt if A. Initiation of a nonselective beta blocker such as
medical therapy fails propranolol
B. Initiation of daily subcutaneous octreotide injections
VII-94. You are asked to consult on a 62-year-old white C. Liver transplantation
woman with pruritus for 4 months. She has noted progres- D. Repeated variceal ligation if the initial bleed was con-
sive fatigue and a 5-lb weight loss. She has intermittent trolled with endoscopic variceal ligation
nausea but no vomiting and denies changes in her bowel E. Transjugular intrahepatic portosystemic shunt
habits. There is no history of prior alcohol use, blood trans-
fusions, or illicit drug use. The patient is widowed and had VII-98. Which of the following statements is true about
two heterosexual partners in her lifetime. Her past medical alcoholic cirrhosis?
history is significant only for hypothyroidism, for which
A. Alcoholic cirrhosis is characterized by predomi-
she takes levothyroxine. Her family history is unremark-
nantly large (>2 cm) nodules in the liver.
able. On examination, she is mildly icteric. She has spi-
B. If patients with alcoholic cirrhosis are able to stop
der angiomata on her torso. You palpate a nodular liver
drinking alcohol, their 5-year survival improves.
edge 2 cm below the right costal margin. The remainder of
C. It is the most common cause of cirrhosis in the
the examination is unremarkable. A right upper quadrant
United States.
ultrasound confirms your suspicion of cirrhosis. You order
D. Parenterally administered tumor necrosis factor
a complete blood count and a comprehensive metabolic
inhibitors have emerged as an important treatment
panel. What is the most appropriate next test?
option that can reduce mortality in patients with
alcoholic cirrhosis.

497
VII-99. A 63-year-old female with primary biliary cirrhosis pain, the patient had a right upper quadrant ultrasound
has refractory ascites that is managed with serial large vol- that demonstrated the presence of gallstones. Following
SECTION VII

ume paracentesis. She presents to the emergency depart- treatment of H. pylori, her symptoms have resolved. She
ment the night after an outpatient large volume tap with is requesting your opinion regarding whether treatment is
altered mental status. On arrival her laboratory studies required for the finding of gallstone disease. On review of
are notable for a serum creatinine of 2.7 mg/dL (baseline the ultrasound report, there were numerous stones in the
0.9 2 weeks ago) and a serum sodium of 127 mEq/L. Her gallbladder, including in the neck of the gallbladder. The
urine output is low. You suspect possible hepatorenal syn- largest stone measures 2.8 cm. What is your advice to the
drome. All of the following would be potential next steps patient regarding the risk of complications and the need
Disorders of the Gastrointestinal System

in her evaluation and management EXCEPT: for definitive treatment?


A. Administer IV albumin to restore effective circulat- A. Given the size and number of stones, prophylactic
ing volume cholecystectomy is recommended.
B. Look for alternative causes of renal failure B. No treatment is necessary unless the patient develops
C. Referral for liver transplantation if creatinine contin- symptoms of biliary colic frequent severe enough to
ues to worsen interfere with the patient’s life.
D. Start furosemide and spironolactone to increase her C. The only reason to proceed with cholecystectomy
urine output is the development of gallstone pancreatitis or
E. Start midodrine and octreotide to try to reduce cholangitis.
splanchnic vasodilation and improve renal perfusion D. The risk of developing acute cholecystitis is about
5–10% per year.
VII-100. Which of the following patients is the highest pri- E. Ursodeoxycholic acid should be given at a dose of
ority for liver transplantation? 10–15 mg/kg daily for a minimum of 6 months to
A. A 24-year-old woman with cirrhosis due to autoim- dissolve the stones.
mune hepatitis; she has been on the transplant list for
VII-103. A 62-year-old man has been hospitalized in inten-
2 months and now has an elevated bilirubin, interna-
sive care for the past 3 weeks following an automobile
tional normalized ratio (INR), and creatinine.
accident resulting in multiple long bone fractures and
B. A 38-year-old woman with chronic hepatitis C and
acute respiratory distress syndrome. He has been slowly
normal bilirubin and INR.
improving but remains on mechanical ventilation. He is
C. A 49-year-old man with alcoholic cirrhosis who has
now febrile and hypotensive requiring vasopressors. He is
been on the transplant list for 6 months; he has had
being treated empirically with cefepime and vancomycin.
two esophageal variceal bleeds.
Multiple blood cultures are negative. He has no new infil-
D. A 59-year-old man with a history of hyperlipidemia
trates or increasing secretions on chest radiograph. His
who was admitted to the intensive care unit 2 days
laboratory studies demonstrated a rise in his liver func-
ago with fulminant hepatic failure due to mistakenly
tion tests, bilirubin, and alkaline phosphatase. Amylase
ingesting Amanita mushrooms from his lawn.
and lipase are normal. A right upper quadrant ultrasound
E. A 64-year-old woman with primary hepatocellular
shows sludge in the gallbladder but no stones. The bile
carcinoma admitted to the hospital with acute renal
duct is not dilated. What is the next best step in the evalu-
failure.
ation and treatment of this patient?
VII-101. Which of the following statements regarding liver A. Discontinue cefepime
transplantation is true? B. Initiate treatment with clindamycin
A. ABO incompatibility is an absolute contraindication C. Initiate treatment with metronidazole
to transplant. D. Perform hepatobiliary scintigraphy
B. Hepatocellular carcinoma is one of several conditions E. Refer for exploratory laparotomy
that receive disease-specific model for end-stage liver
VII-104. A 41-year-old woman presents to your clinic with
disease exceptions.
a week of jaundice. She notes pruritus, icterus, and dark
C. HIV infection is a contraindication to transplant.
urine. She denies fever, abdominal pain, or weight loss.
D. Living donor transplants pose very little risk to the
The examination is unremarkable except for yellow dis-
donor.
coloration of the skin. Total bilirubin is 6.0 mg/dL, and
E. Preformed cytotoxic human leukocyte antigen anti-
direct bilirubin is 5.1 mg/dL. Aspartate aminotransferase
bodies are a contraindication to transplantation.
is 84 IU/L, alanine aminotransferase is 92 IU/L, and alka-
VII-102. A 44-year-old woman is evaluated for complaints line phosphatase is 662 IU/L. CT scan of the abdomen is
of abdominal pain. She describes the pain as a postpran- unremarkable. Right upper quadrant ultrasound shows
dial burning pain. It is worse with spicy or fatty foods and a normal gallbladder but does not visualize the common
is relieved with antacids. She is diagnosed with a gastric bile duct. What is the most appropriate next management
ulcer and is treated appropriately for Helicobacter pylori. step?
During the course of her evaluation for her abdominal

498
WWW.BOOKBAZ.IR
A. Antibiotics and observation A. Esophageal carcinoma
B. Endoscopic retrograde cholangiopancreatography B. Intestinal obstruction

SECTION VII
C. Hepatitis serologies C. Mumps
D. Hepatobiliary iminodiacetic acid scan D. Pregnancy
E. Serologies for antimitochondrial antibodies E. Renal Failure

VII-105. All of the following are contributing factors to cho- VII-109. Which of the following statements about imaging
lesterol stone formation EXCEPT: of the pancreas is true?
A. An excess of biliary cholesterol in relation to bile A. Abdominal CT has no role in the diagnosis of acute

QUESTIONS
acids and phospholipids pancreatitis.
B. Increased biliary secretion of cholesterol B. Endoscopic ultrasound (EUS) and magnetic reso-
C. Gallbladder hypomotility nance cholangiopancreatography have largely
D. Nucleation of cholesterol monohydrate crystals replaced endoscopic retrograde cholangiopancrea-
E. Rapid weight gain through a high-fat diet tography in the diagnostic evaluation of pancreatic
disease.
VII-106. A previously healthy 46-year-old female presents C. EUS is not helpful in the diagnosis of chronic
with 12 hours of right upper quadrant pain and nausea. pancreatitis.
The pain occasionally radiates to her right scapula. She D. Plane radiographs of the abdomen are com-
has exquisite right upper quadrant tenderness on deep monly used in the evaluation of acute and chronic
palpation. Laboratory studies show a mild elevation in her pancreatitis.
serum alkaline phosphatase. A right upper quadrant ultra- E. One advantage of pancreatic ultrasound is that intes-
sound shows stones in the bladder and a thickened gall- tinal bowel gas does not interfere with the ability to
bladder wall consistent with acute calculous cholecystitis. obtain images.
All of the following statements about cholecystectomy are
true EXCEPT: VII-110. Which of the following statements regarding pan-
creatic enzyme secretion is true?
A. Five to ten percent of patients who undergo elective
cholecystectomy develop diarrhea. A. Acetylcholine is an important neurotransmitter that
B. Atelectasis and subphrenic abscesses may occur after stimulates pancreatic enzyme secretion.
cholecystectomy. B. All pancreatic enzymes have pH optima in the acidic
C. Cholecystectomy provides near total relief of symp- range.
toms in up to 75–90% of patients. C. All pancreatic enzymes are secreted in their active
D. The complication rate is higher in patients who forms.
undergo early cholecystectomy as opposed to delayed D. Cholecystokinin receptors on human pancreatic aci-
cholecystectomy (>6 weeks). nar cells are important in pancreatic enzyme secretion.
E. Undetected common bile duct stones are left behind E. Enterokinase found in the duodenal mucosa acti-
in 1–5% of patients. vates vasoactive intestinal peptide which then cleaves
zymogens into active proteases.
VII-107. A 46-year-old woman presents with a 1-week his-
tory of right upper quadrant pain, intermittent fever, VII-111. A 45-year-old woman with a known history of
and nausea. Her initial labs are notable for a markedly cholelithiasis is admitted to the hospital with severe mid-
elevated bilirubin, mild elevations in her transaminases, epigastric pain, fever to 38.5°C (101.3°F), tachycardia to
and a modest elevation in her alkaline phosphatase. 110 beats/min, and a blood pressure of 100/50. Her exami-
Ultrasound shows no gallstones. Magnetic resonance nation shows a diffusely tender abdomen with guarding.
cholangiopancreatography reveals a beaded appearance Radiographs show an abdominal ileus with no free air.
of her intrahepatic and extrahepatic bile ducts due to Labs are notable for a hemoglobin of 15 g/dL and eleva-
multiple discrete strictures. Which of the following lab tions of amylase and lipase. Which of the following state-
tests would be most helpful in understanding the cause ments regarding this patient’s likely diagnosis is true?
of her current disease?
A. Elevated lipase is more specific than elevated amylase
A. Hepatitis C antibody for the diagnosis of acute pancreatitis.
B. Procalcitonin B. Hypercalcemia occurs in >75% of cases of acute
C. Serum IgG-4 levels pancreatitis.
D. Serum triglycerides C. Magnitude of lipase elevation above normal is cor-
E. Urine histoplasma antigen related with the severity of acute pancreatitis.
D. Serum amylase levels will remain elevated for up to
VII-108. All of the following disorders may be associated 30 days after the resolution of acute pancreatitis.
with a rise in serum amylase levels EXCEPT: E. The combination of elevated serum amylase and
metabolic acidosis (pH <7.32) has a >90% positive
predictive value for acute pancreatitis.

499
VII-112. A 27-year-old woman is admitted to the hospital tachycardia with a pulse of 110 beats/min and a fever up
with acute-onset severe right upper quadrant pain that to 38.1°C (100.6°F). After receiving 3 L of IV lactated ring-
SECTION VII

radiates to the back. The pain is constant and not relieved ers solution over the first 24 hours, her hematocrit drops
with eating or bowel movements. Her labs show marked to 38. Her heart rate and temperature have also improved.
elevation in amylase and lipase, and acute pancreatitis is What is the significance of the drop in hematocrit?
diagnosed. Which of the following is the best first test to
A. Current fluid management should be continued as
demonstrate the etiology of her pancreatitis?
her drop in hematocrit is evidence of effective fluid
A. Right upper quadrant ultrasound resuscitation.
B. Serum alcohol level B. She has likely developed hemorrhage from severe
Disorders of the Gastrointestinal System

C. Serum triglyceride level necrotizing pancreatitis and should undergo urgent


D. Technetium hepatobiliary iminodiacetic acid scan CT scanning of the abdomen.
E. Urine drug screen C. She should be evaluated for hemolysis as a cause of
the drop in hematocrit.
VII-113. A 58-year-old female with known alcohol abuse to D. She should receive two units of packed red blood
the hospital with abdominal pain after a drinking binge. cells to prevent organ dysfunction from hypoperfu-
Her symptoms have been present for 3 days, and she has sion related to evolving anemia.
persisted to drink heavily. She now has persistent vomit- E. She should undergo endoscopic ultrasound to look
ing and feels dizzy on standing. On examination, she has for an evolving pancreatic pseudocyst.
severe epigastric and right upper quadrant tenderness and
decreased bowel sounds, and she appears uncomfortable. VII-116. A 56-year-old male presents to the emergency
She has a blue-red discoloration of the flanks bilaterally department with 1–2 days of worsening abdominal pain,
but denies any recent trauma. What is the significance of fever, tachycardia and shortness of breath. He denies any
this finding? alcohol use and has no known history of gallstones. His
initial labs are notable for a markedly elevated amylase and
A. Abdominal plain film is likely to show pancreatic
lipase. He is given 2 L of IV isotonic fluids and is started
calcification.
on empiric antibiotics. CT of the chest, abdomen, and pel-
B. CT of the abdomen is likely to show severe necrotiz-
vis reveal severe necrotizing pancreatitis, as well as bilat-
ing pancreatitis with hemoperitoneum.
eral alveolar infiltrates with basilar atelectasis. He requires
C. Endoscopic ultrasound is likely to reveal a pancreatic
intubation and mechanical ventilation for evolving respir-
mass.
atory failure. All cultures are negative. Which of the fol-
D. Right upper quadrant ultrasound will likely show
lowing best explains the etiology of his respiratory failure?
acute cholecystitis.
E. Transvaginal ultrasound will likely show a ruptured A. He has developed acute respiratory distress syn-
ectopic pregnancy. drome (ARDS) from severe pancreatitis.
B. He has developed heart failure from overly aggressive
VII-114. A 42-year-old male is admitted with severe epigas- fluid resuscitation.
tric pain radiating to the back. You suspect acute pancrea- C. He has developed pulmonary hemorrhage related to
titis. All of the following are accepted diagnostic criteria coagulopathy from sepsis.
for acute pancreatitis EXCEPT: D. He likely has culture-negative pneumonia and devel-
A. Confirmatory findings of acute pancreatitis on cross- oped ARDS from direct lung injury.
sectional abdominal imaging E. He likely has sepsis from necrotizing pancreatitis and
B. PaO2 <60 developed ARDS from the resulting bacteremia.
C. Threefold or greater elevation in serum lipase and/or
VII-117. Which of the following statements regarding
amylase
enteral feeding in acute pancreatitis is true?
D. Typical abdominal pain in the epigastrium that may
radiate to the back A. A patient with persistent evidence of pancreatic
necrosis on CT 2 weeks after acute presentation
VII-115. A 45-year-old female with known gallstones pre- should be maintained on bowel rest.
sents to the hospital with 2 days of mid-epigastric tender- B. All patients with elevations of amylase and lipase and
ness radiating to her back, as well as nausea and vomiting. CT evidence of pancreatitis should be fasted until
Her serum lipase is elevated to four times the upper limit amylase and lipase normalize.
of normal and abdominal ultrasound reveals gallstones C. Enteral feeding has been demonstrated to have
and findings consistent with acute pancreatitis. Her initial fewer infectious complications than total parenteral
hematocrit is 48 and her vital signs are notable for sinus

500
WWW.BOOKBAZ.IR
nutrition in the management of patients with acute VII-118. A 25-year-old woman with cystic fibrosis is diag-
pancreatitis. nosed with chronic pancreatitis. She is at risk for all of the

SECTION VII
D. Patients requiring surgical removal of infected pan- following complications EXCEPT:
creatic pseudocysts should be treated with total par-
A. Vitamin B12 deficiency
enteral nutrition.
B. Vitamin A deficiency
E. Total parenteral nutrition has been shown to maintain
C. Pancreatic carcinoma
integrity of the intestinal tract in acute pancreatitis.
D. Niacin deficiency
E. Steatorrhea

ANSWERS
ANSWERS

VII-1. The answer is E. (Chap. 314) The colon is primarily responsible for stool dehydration,
decreasing volumes from 1000–1500 mL to 100–200 mL. The stomach triturates and
mixes the food bolus with pepsin and acid. The stomach also secretes intrinsic factor,
which is necessary for vitamin B12 absorption. Gastric acid sterilizes the upper gut. Pan-
creatic juice contains enzymes for carbohydrate, protein, and fat digestion as well as
bicarbonate to optimize the pH for enzyme activation.

VII-2. The answer is C. (Chap. 314) Pernicious anemia is associated with little or no gastric acid
due to a lack of intrinsic factor. This is caused by autoantibodies that damage the gastric
parietal cells. Gastrin G-cell hyperplasia results in increased gastrin levels, which leads
to gastric acid hypersecretion. About 50% of G-cell hyperplasia is related to Helicobacter
pylori infection and can be treated with eradication of the bacterium. Retained antrum
syndrome is a rare postgastrectomy syndrome following a Billroth-II procedure in which
the antrum has not been entirely removed. Some patients with duodenal ulcers have acid
hypersecretion. Zollinger-Ellison syndrome is caused by a tumor (usually in the pancreas
or duodenum) that secretes gastrin and therefore causes gastric acid hypersecretion.

VII-3. The answer is D. (Chap. 314) Mesenteric ischemia, biliary colic, and neoplasms most
commonly cause noninflammatory visceral pain. Peptic ulcer, appendicitis, diverticuli-
tis, inflammatory bowel disease, pancreatitis, cholecystitis, and infectious enterocolitis
are all painful inflammatory diseases.

VII-4. The answer is A. (Chap. 314) Diarrhea from malabsorption usually improves with fast-
ing, whereas secretory diarrhea persists without oral intake. Sudden awakening from
sound sleep by pain suggests organic rather than functional disease. Symptoms from
mechanical obstruction, ischemia, inflammatory bowel disease, and functional bowel
disorders are worsened by meals. Ulcer pain occurs intermittently over weeks to months,
whereas biliary colic has a sudden onset and lasts up to several hours. Ulcer symptoms
may be relieved by eating or antacids.

VII-5. The answer is B. (Chap. 315) Since this patient is over 50 years old, even without a fam-
ily history, screening for colorectal cancer is recommended. Colonoscopy is the gold
standard for imaging the colonic mucosa. The cecum is reached in >95% of cases and
the terminal ileum can often be examined. Colonoscopy has greater sensitivity than
barium enema for colitis, polyps, and cancer. CT colonography rivals the accuracy of
colonoscopy for detection of some polyps and cancer, although it is not as sensitive for
the detection of flat lesions, such as serrated polyps. Flexible sigmoidoscopy is similar
to colonoscopy, but it visualizes only the rectum and a variable portion of the left colon,
typically to 60 cm from the anal verge.

501
SECTION VIII
Rheumatology and Immunology

QUESTIONS

DIRECTIONS: Choose the one best response to each VIII-4. Which of the following diseases has a strong genetic
question. association with a particular class I major histocompatibility
complex alleles?
A. Ankylosing spondylitis
VIII-1. A 55-year-old woman who was previously incar- B. Celiac disease
cerated has an area of induration that measures 15 mm C. Huntington disease
72 hours after a skin test with tuberculin purified protein C. Rheumatoid arthritis
derivative (PPD). A positive PPD skin test for Mycobac- D. Type 1 diabetes
terium tuberculosis represents which type of immune
reaction? VIII-5. Which of the following statements best describes the
function of proteins encoded by the human major histo-
A. Cytotoxic reaction of antibody compatibility complex I and II genes?
B. Delayed-type hypersensitivity reaction
C. Immediate-type hypersensitivity reaction A. Activation of the complement system
D. Immune complex formation B. Binding to cell surface receptors on granulocytes and
macrophages to initiate phagocytosis
VIII-2. All of the following are key features of the innate C. Nonspecific binding of antigen for presentation to
immune system EXCEPT: T cells
D. Specific antigen binding in response to B-cell activa-
A. Exclusively a feature of vertebrate animals
tion to promote neutralization and precipitation
B. Important cells include macrophages and natural
killer lymphocytes VIII-6. All of the following statements regarding primary
C. Nonrecognition of benign foreign molecules or immunodeficiency disorders are true EXCEPT:
microbes
D. Recognition by germline-encoded host molecules A. Infections of the upper or lower respiratory tract sug-
E. Recognition of key microbe virulence factors but no gest a defective antibody response.
recognition of self-molecules B. Most are diagnosed by the presence of recurrent or
unusually severe infections.
VIII-3. A 29-year-old man with episodic abdominal pain C. Recurrent infections due to Candida species suggest
and stress-induced edema of the lips, tongue, and occa- impaired T-cell immunity.
sionally larynx is likely to have low functional or absolute D. They are typically genetic diseases with Mendelian
levels of which of the following proteins? inheritance.
E. While most aspects of the immune system may be
A. C1 esterase inhibitor
involved, innate immunity is not affected by these
B. C5A (complement cascade)
disorders.
C. Cyclooxygenase
D. IgE
E. T-cell receptor, α chain

543
VIII-7. A 19-year-old college freshman comes to the uni- VIII-8. A 35-year-old woman from Virginia presents to your
SECTION VIII

versity clinic complaining of tender, painful skin lesions clinic with worsening sneezing, rhinorrhea, and nasal itch-
in his axilla (Figure VIII-7). He reports that he has had ing, which has recurred yearly since she was an adolescent
similar episodes throughout his life for which he receives starting in March and increasing during April and May.
antibiotics. He has a lab printout from his last episode that Based on this history she has an elevated risk of having all
reports a positive culture for Serratia marcescens. All of the of the following additional conditions EXCEPT:
following statements regarding this patient and his likely
A. Asthma
diagnosis are true EXCEPT:
B. Chronic bilateral sinusitis
Rheumatology and Immunology

C. Food allergies
D. Psoriasis
E. Urticaria

VIII-9. A 28-year-old woman seeks evaluation from her pri-


mary care doctor for recurrent episodes of hives and states
that she is “allergic to cold weather.” She reports that for
more than 10 years she would develop areas of hives when
exposed to cold temperatures, usually on her arms and
legs. She has never sought evaluation previously and states
that, over the last several years, the occurrence of the hives
has become more frequent. Other than cold exposure, she
can identify no other triggers for development of hives.
She has no history of asthma or atopy. She denies food
intolerance. Her only medication is oral contraceptive
pills, which she has taken for 5 years. She lives in a single-
family home that was built 2 years ago. On examination,
she develops a linear wheal after being stroked along her
forearm with a tongue depressor. On placing her hand in
cold water, her hand becomes red and swollen. In addition,
there are several areas with a wheal and flare reaction on
the arm above the area of cold exposure. What is the next
step in the management of this patient?
A. Assess for the presence of antithyroglobulin and anti-
microsomal antibodies
B. Check C1 inhibitor levels
C. Discontinue the oral contraceptive pills
D. Treat with cetirizine 10 mg daily
E. Treat with cyproheptadine 8 mg daily

VIII-10. A 45-year-old man from Arkansas with a history


of hypertension that is treated with lisinopril and colorec-
FIGURE VIII-7 Reproduced with permission from Wolff K et al: tal cancer presents to your clinic for a follow-up appoint-
Fitzpatrick’s Color Atlas and Synopsis of Clinical Dermatology, 8th ed. ment after a recent emergency room visit. He tells you he
New York: McGraw Hill, 2017. was recently diagnosed with an episode of anaphylaxis. He
presented to an emergency room with laryngeal edema
and diffuse urticaria and was treated successfully with
epinephrine. On further history, he tells you more about
A. Human stem cell transplantation is curative. his recent diagnosis of colorectal cancer for which he has
B. Infections with catalase-negative organisms are received several doses of cetuximab. He was also sick with
typical. shortness of breath and cough 2 weeks prior to the ana-
C. Prophylactic use of trimethoprim/sulfamethoxazole phylactic episode. He was evaluated with a CT of the chest
is effective in reducing risk of bacterial infections. with IV contrast, diagnosed with pneumonia, and treated
D. The disease is caused by defective production of reac- with ceftriaxone. Immediately before his episode of ana-
tive oxygen species in phagolysosomes. phylaxis, he had been enjoying a steak dinner for his wife’s
E. The disease is most likely transmitted by X-linked birthday. What is the antigen for the IgE antibodies that
recessive inheritance. most likely caused his anaphylactic reaction?

544
WWW.BOOKBAZ.IR
A. Alpha-1,3-galactose (alpha-gal) VIII-13. A 25-year-old man is admitted to the intensive care

SECTION VIII
B. Ceftriaxone unit with hypoxemia. His partner tells the intensive care
C. Iodinated contrast team that he has been coughing up blood for the last 2 days
D. Lisinopril in increasing amounts. On laboratory studies his creati-
nine is markedly elevated. He is ultimately diagnosed with
VIII-11. You are working in the emergency department Goodpasture syndrome. What is the immune mechanism
when a 3-year-old boy arrives by ambulance. He was eat- leading to organ damage in Goodpasture syndrome?
ing tonight when he suddenly started wheezing, coughing,
and then became progressively less responsive. His parents A. Antibody-dependent cellular cytotoxicity

QUESTIONS
are certain he did not aspirate. On arrival, his blood pres- B. Complement-activating autoantibody
sure is low, and he is working hard to breathe. You auscul- C. Inactivating autoantibody
tate tight wheezes bilaterally. You accurately diagnose him D. Stimulating autoantibody
with anaphylaxis and initiate appropriate therapy. Which E. T-cell–mediated cellular cytotoxicity
of the following statements regarding anaphylaxis is true?
VIII-14. Rheumatic fever develops due to an autoimmune
A. An atopic history is a risk factor for anaphylaxis to process. Which of the following mechanisms of autoim-
penicillin therapy. munity is primarily responsible for the development of
B. Onset of anaphylaxis is most often 1–2 hours after rheumatic fever?
antigen exposure.
A. Endocrine abnormalities
C. IV glucocorticoids are effective for acute anaphylaxis.
B. Increased B-cell function
D. Older age is associated with improved outcomes in
C. Intrinsic cytokine imbalance
anaphylaxis.
D. Increased T-cell help due to cytokine stimulation
E. The failure to use epinephrine within the first
E. Molecular mimicry
20 minutes of symptoms is a risk factor for death due
to anaphylaxis. VIII-15. A 60-year-old Caucasian man with a history of pso-
riasis presents to the clinic for a follow-up appointment.
VIII-12. A 60-year-old man presents to the clinic with inter-
He has been treated with infliximab (a tumor necrosis
mittent flushing, diarrhea, headaches, and a new skin rash
factor inhibitor) for about a year. Since his last infusion of
of several months’ duration. He has noticed that his symp-
infliximab, he has been experiencing aching in his muscles
toms of diarrhea and flushing tend to worsen if he takes
and joints and a new facial rash. On examination you note
ibuprofen to treat his headaches. On examination, he has
a raised erythematous malar rash, but there is no synovitis
a palpable spleen and cervical lymphadenopathy. Serum
in any joints and there is no weakness on neurologic exam-
tryptase is elevated at 35 ng/mL. His skin rash is shown
ination. Laboratory testing shows a positive anti-nuclear
in Figure VIII-12. A diagnostic skin biopsy is performed.
antibody at a titer of 1:320. Which specific autoantibody is
What histological findings are most likely to be noted on
most likely to be present?
the skin biopsy?
A. Anti-β-2-glycoprotein
A. Interface dermatitis
B. Anti-cyclic citrullinated protein antibodies
B. Multifocal dense infiltrates of mast cells
C. Anti-dsDNA antibodies
C. Neutrophilic dermal infiltrate
D. Anti-histone antibodies
D. Noncaseating granulomas
E. Anti-thyroid peroxidase antibodies
E. Spongiotic changes in the epidermis

FIGURE VIII-12 Reproduced with permission from Lichtman MA et al: Lichtman’s Atlas of Hematology 2016. New York: McGraw Hill, 2017.

545
VIII-16. All of the following are predisposing factors for sys- A. Antiphospholipid antibodies are generally directed
SECTION VIII

temic lupus erythematosus EXCEPT: against negatively charged phospholipids including


cardiolipin, phosphocholine, and phosphatidylserine.
A. C1 esterase inhibitor deficiency
B. Patients who are positive for the lupus anticoagulant
B. Female sex
will have an elevated activated partial thromboplas-
C. HLA-DR alleles
tin time in vitro.
D. Smoking
C. Patients with antiphospholipid antibodies may test
E. Ultraviolet light
false positive for syphilis.
VIII-17. A 32-year-old woman with long-standing diagnosis D. The majority of patients with systemic lupus erythe-
Rheumatology and Immunology

of systemic lupus erythematosus is evaluated by her rheu- matosus and antiphospholipid antibodies develop
matologist as routine follow-up. A new cardiac murmur clinical manifestations of antiphospholipid syndrome.
is heard, and an echocardiogram is ordered. She is feeling
VIII-20. A 27-year-old woman is admitted to the intensive
well and has no fevers, weight loss, or pre-existing cardiac
care unit after recent delivery of a full-term infant 3 days
disease. A vegetation on the mitral valve is demonstrated.
prior. The patient was found to have right hemiparesis and
Which of the following statements is true?
a blue left hand. Physical examination is also notable for
A. Blood cultures are unlikely to be positive. livedo reticularis. Her laboratories were notable for a white
B. Glucocorticoid therapy has been proven to lead to blood cell count of 10.2/μL, hematocrit of 35%, and platelet
improvement in this condition. count of 13,000/μL. Her blood urea nitrogen is 36 mg/dL,
C. Pericarditis is frequently present concomitantly. and her creatinine is 2.3 mg/dL. Although this pregnancy
D. The lesion has a low risk of embolization. was uneventful, the three prior pregnancies resulted in early
E. The patient has been surreptitiously using injection losses. A peripheral smear shows no evidence of schisto-
drugs. cytes. Which of the following laboratory studies will best
confirm the underlying etiology of her presentation?
VIII-18. A 45-year-old African American woman with sys-
temic lupus erythematosus (SLE) presents to the emer- A. Anticardiolipin antibody panel
gency department with complaints of headache and B. Antinuclear antibody
fatigue. Her prior manifestations of SLE have been arthral- C. Doppler examination of her left arm arterial tree
gias, hemolytic anemia, malar rash, and mouth ulcers, and D. Echocardiography
she is known to have high titers of antibodies to double- E. MRI of her brain
stranded DNA. She currently is taking prednisone, 5 mg
VIII-21. A 28-year-old woman comes to the emergency
daily, and hydroxychloroquine, 200 mg daily. On presen-
department complaining of 1 day of worsening right leg
tation, she is found to have a blood pressure of 190/110
pain and swelling. She drove in a car 8 hours back from a
with a heart rate of 98 beats/min. A urinalysis shows 25 red
hiking trip 2 days ago and then noticed some pain in the
blood cells (RBCs) per high-powered field with 2+ protein-
leg. At first she thought it was due to exertion, but it has
uria. No RBC casts are identified. Her blood urea nitrogen is
worsened over the day. Her only past medical history is
88 mg/dL, and creatinine is 2.6 mg/dL (baseline 0.8 mg/dL).
related to difficulty getting pregnant, with two prior spon-
She has not previously had renal disease related to SLE and
taneous abortions. Her physical examination is notable for
is not taking nonsteroidal anti-inflammatory drugs. She
normal vital signs and heart and lung examination. Her
denies any recent illness, decreased oral intake, or diarrhea.
right leg is swollen from the mid-thigh down and is tender.
What is the most appropriate next step in the management
Doppler studies demonstrate a large deep venous throm-
of this patient?
bosis in the femoral and iliac veins extending into the pel-
A. Initiate cyclophosphamide, 500 mg/m2 body surface vis. Laboratory studies on admission prior to therapy show
area IV, and plan to repeat monthly for 3–6 months normal electrolytes, normal white blood cell and platelet
B. Initiate hemodialysis counts, normal prothrombin time, and an activated par-
C. Initiate high-dose steroid therapy (IV methylpred- tial thromboplastin time three times the normal value. Her
nisolone, 1000 mg daily for three doses, followed by pregnancy test is negative. Low-molecular-weight heparin
oral prednisone, 1 mg/kg daily) and mycophenolate therapy is initiated in the emergency department. Subse-
mofetil, 2 g daily quent therapy should include which of the following?
D. Initiate plasmapheresis
A. Rituximab 375 mg/m2 per week for 4 weeks
E. Withhold all therapy until renal biopsy is performed
B. Warfarin with international normalized ratio (INR)
VIII-19. All of the following statements regarding antiphos- goal of 2.0–3.0 for 3 months
pholipid antibodies are true EXCEPT: C. Warfarin with INR goal of 2.0–3.0 for 12 months
D. Warfarin with INR goal of 2.5–3.5 for life
E. Warfarin with an INR goal of 2.5–3.5 for 12 months
followed by daily aspirin for life

546
WWW.BOOKBAZ.IR
VIII-22. When compared with patients with rheumatoid A. Bilateral interstitial infiltrates

SECTION VIII
arthritis (RA) who do not have anti-cyclic citrullinated B. Bronchiectasis
protein (CCP) antibodies, patients with RA who have anti- C. Lobar infiltrate
CCP antibodies: D. Solitary pulmonary nodule
E. Unilateral pleural effusion
A. Are less likely to develop extra-articular manifesta-
tions of RA (e.g., vasculitis) VIII-27. Which of the following is the earliest plain radio-
B. Are less likely to have a history of smoking graphic finding of rheumatoid arthritis?
C. Develop fewer subchondral bone erosions on imag-
A. Juxta-articular osteopenia

QUESTIONS
ing over their disease course
D. Have a higher prevalence of shared epitope (HLA- B. No abnormality
DRB1) risk alleles C. Soft tissue swelling
E. Have lower scores for physical disability D. Subchondral erosions
E. Symmetric joint space loss
VIII-23. A 65-year-old woman with a 10-year history of
untreated rheumatoid arthritis presents to your clinic with VIII-28. All of the following agents have been shown to have
worsening joint pain and malaise over the last 6 months. On disease-modifying antirheumatic drug efficacy in patients
examination of her joints, she has swan neck deformities with rheumatoid arthritis EXCEPT:
and ulnar deviation. Her skin examination demonstrates A. Infliximab
rheumatoid nodules in the olecranon bursa bilaterally. On B. Leflunomide
abdominal examination, she has splenomegaly. Labora- C. Methotrexate
tory studies show neutropenia, elevated C-reactive pro- D. Naproxen
tein, and anemia. She is diagnosed with Felty syndrome. E. Rituximab
What type of hematologic malignancy or lymphoprolifera-
tive disorder may present similarly to Felty syndrome in VIII-29. Which of the following findings is the most com-
patients with rheumatoid arthritis? mon clinical presentation of acute rheumatic fever?
A. Acute myeloid leukemia A. Carditis
B. Chronic lymphocytic leukemia B. Chorea
C. Essential thrombocytosis C. Erythema marginatum
D. Polycythemia vera D. Polyarthritis
E. T-cell large granular lymphocyte leukemia E. Subcutaneous nodules

VIII-24. Patients with rheumatoid arthritis are at higher risk VIII-30. A 19-year-old recent immigrant from Ethiopia
for all of the following health conditions than the general comes to your clinic to establish primary care. She cur-
population EXCEPT: rently feels well. Her past medical history is notable for a
recent admission to the hospital for new-onset atrial fibril-
A. Colorectal cancer
lation. As a child in Ethiopia, she developed an illness that
B. Coronary artery disease
caused uncontrolled flailing of her limbs and tongue lasting
C. Hypoandrogenism
approximately 1 month. She also has had three episodes of
D. Lymphoma
migratory large-joint arthritis during her adolescence that
E. Osteoporosis
resolved with pills that she received from the pharmacy.
VIII-25. Which of the following is the most frequent site of She is currently taking metoprolol and warfarin and has
joint involvement in established rheumatoid arthritis? no known drug allergies. Physical examination reveals an
irregularly irregular heart beat with normal blood pres-
A. Distal interphalangeal joint sure. Her point of maximal impulse is most prominent
B. Hip at the midclavicular line and is normal in size. An early
C. Knee diastolic rumble and 3/6 holosystolic murmur are heard at
D. Spine the apex. A soft early diastolic murmur is also heard at the
E. Wrist left third intercostal space. You refer her to a cardiologist
for evaluation of valve replacement and echocardiography.
VIII-26. In patients with established rheumatoid arthritis,
What other intervention might you consider at this time?
all of the following pulmonary radiographic findings may
be explained by their rheumatologic condition EXCEPT: A. Daily aspirin
B. Daily doxycycline
C. Low-dose corticosteroids
D. Monthly penicillin G injections
E. Penicillin G injections as needed for all sore throats

547
VIII-31. Most of the manifestations of acute rheumatic weight loss and night sweats. Physical examination shows
SECTION VIII

fever present approximately 3 weeks after the precipitating parotid gland swelling bilaterally and decreased salivary
group A streptococcal infection. Which manifestation may pooling, but no rashes, joint swelling, or other findings.
present several months after the precipitating infection? Laboratory testing is negative for anti-nuclear antibodies,
Ro/SS-A antibodies, and La/SS-B antibodies. Chest radio-
A. Chorea
graph shows no infiltrates or lymphadenopathy. What is
B. Erythema marginatum
the most likely diagnosis?
C. Fever
D. Polyarthritis A. HIV infection
Rheumatology and Immunology

E. Subcutaneous nodules B. IgG4-related disease


C. Primary Sjögren syndrome
VIII-32. A 60-year-old woman with long-standing sclero- D. Rheumatoid arthritis with secondary Sjögren
derma (Raynaud phenomenon, diffuse skin disease, and syndrome
pulmonary fibrosis) presents to the clinic with new weight E. Sarcoidosis
loss. She has also been noting increased lower abdomi-
nal pain, bloating, and diarrhea recently. She denies any VIII-37. A 57-year-old woman with depression and chronic
heartburn or dysphagia. She has not had melena or hema- migraine headaches reports several years of dry mouth and
tochezia. Which of the following mechanisms related to dry eyes. Her primary complaint is that she can no longer
her scleroderma could be contributing to her symptoms? eat her favorite crackers, although she does report photo-
sensitivity and eye burning on further questioning. She
A. Gastric antral vascular ectasia
has no other associated symptoms. Examination shows
B. Gastroesophageal reflux disease
dry, erythematous, sticky oral mucosa. All of the following
C. Impaired intestinal motility
tests are likely to be positive in this patient EXCEPT:
D. Small bowel bacterial overgrowth
E. C and D A. La/SS-B antibody
B. Ro/SS-A antibody
VIII-33. Patients with scleroderma are at higher risk for all C. Schirmer I test
of the following complications in the cardiovascular sys- D. Scl-70 antibody
tem EXCEPT: E. Sialometry
A. Abdominal aortic aneurysm
VIII-38. A patient with primary Sjögren syndrome that was
B. Heart block due to conduction system fibrosis
diagnosed 6 years ago and treated with tear replacement
C. Left ventricular dysfunction
for symptomatic relief notes continued parotid swelling
D. Pulmonary arterial hypertension
for the last 3 months. She has also noted enlarging pos-
E. Raynaud phenomenon
terior cervical lymph nodes. Evaluation shows leukopenia
VIII-34. A patient with a diagnosis of scleroderma who and low C4 complement levels. What is the most likely
has diffuse cutaneous involvement presents with malig- diagnosis?
nant hypertension, oliguria, edema, hemolytic anemia, A. Amyloidosis
and renal failure. You make a diagnosis of scleroderma B. Chronic pancreatitis
renal crisis. Which of the following is the recommended C. HIV infection
treatment? D. Lymphoma
A. Captopril E. Secondary Sjögren syndrome
B. Carvedilol
VIII-39. Which of the following is the most common extrag-
C. Clonidine
landular manifestation of primary Sjögren syndrome?
D. Diltiazem
E. Nitroprusside A. Arthralgias/arthritis
B. Lymphoma
VIII-35. Which of the following conditions is nearly twice as C. Peripheral neuropathy
common in patients with diffuse cutaneous systemic scle- D. Raynaud phenomenon
rosis than in limited cutaneous systemic sclerosis? E. Vasculitis
A. Esophageal involvement
VIII-40. All of the following medications are effective in
B. Pulmonary arterial hypertension
treating both psoriatic arthritis and the skin disease of
C. Pulmonary fibrosis
psoriasis EXCEPT:
D. Raynaud phenomenon
E. Skin involvement A. Adalimumab
B. Infliximab
VIII-36. A 22-year-old man with a history of injection drug C. Rituximab
use presents to the clinic with a new complaint of dry D. Secukinumab
mouth. He denies any eye dryness, photosensitivity, or E. Ustekinumab
foreign body sensation in the eye. He has also experienced

548
WWW.BOOKBAZ.IR
VIII-41. A 30-year-old woman presents to an internal medi- urination. Examination shows inflammatory arthritis of

SECTION VIII
cine clinic for evaluation of joint pain and swelling of the right knee, dactylitis, and normal genitourinary exam-
about 2 years duration accompanied by morning stiffness. ination. He is diagnosed with reactive arthritis. Which
She denies any back or neck pain. Symptoms have primar- of the following was the most likely etiologic agent of his
ily affected her lower extremities. She notes intermittent diarrhea?
swelling in her Achilles tendons, ankles, and knees. She
A. Campylobacter jejuni
denies any psoriasis or family history of skin disease. She
B. Clostridium difficile
denies any gastrointestinal symptoms or preceding infec-
C. Escherichia coli
tious episodes. On physical examination she has right-

QUESTIONS
D. Helicobacter pylori
sided Achilles enthesitis, left ankle synovitis, and dactylitis
E. Shigella flexneri
of the third and fourth fingers of her right hand. Imag-
ing of her sacroiliac joints with MRI shows no erosions or VIII-46. Which of the following definitions best fits the term
bone marrow edema. Laboratory testing shows an elevated enthesitis?
C-reactive protein, negative rheumatoid factor, and nega-
tive anti-cyclic citrullinated protein antibodies. Which of A. A palpable vibratory or crackling sensation elicited
the following is the most likely diagnosis? with joint motion
B. Alteration of joint alignment so that articulating sur-
A. Ankylosing spondylitis faces incompletely approximate each other
B. Peripheral spondyloarthritis C. Inflammation at the site of tendinous or ligamentous
C. Psoriatic arthritis insertion into bone
D. Reactive arthritis D. Inflammation of the periarticular membrane lining
E. Rheumatoid arthritis the joint capsule
E. Inflammation of a saclike cavity near a joint that
VIII-42. Which of the following clinical manifestations can
decreases friction
be seen in the syndrome of synovitis, acne, pustulosis,
hyperostosis, and osteitis? VIII-47. All of the following help distinguish psoriatic
A. Acromegaly arthritis from other joint disorders EXCEPT:
B. Hidradenitis suppurativa A. Dactylitis
C. Plaque psoriasis B. Enthesitis
D. Sternoclavicular osteomyelitis C. Nail pitting
E. B and D D. Presence of diarrhea
E. Shortening of digits
VIII-43. Histocompatibility antigen human leukocyte
antigen-B27 is present in what percentage of North Amer- VIII-48. Which cardiac valvular lesion is most common in
ican patients with ankylosing spondylitis? patients with ankylosing spondylitis?
A. 10% A. Aortic regurgitation
B. 30% B. Mitral regurgitation
C. 50% C. Mitral stenosis
D. 80% D. Pulmonic stenosis
E. 100% E. Tricuspid regurgitation
VIII-44. Which of the following is the most common extra- VIII-49. A 55-year-old man presents to the emergency
articular manifestation of ankylosing spondylitis? department with weight loss, fever, testicular pain, and
A. Anterior uveitis a new rash on his legs. On physical examination, he has
B. Aortic insufficiency abdominal tenderness and raised nonblanching erythe-
C. Inflammatory bower disease matous lesions on his legs. Laboratory studies show an
D. Pulmonary fibrosis elevated erythrocyte sedimentation rate and C-reactive
E. Third-degree heart block protein as well as a positive hepatitis B surface antigen. He
undergoes a surgical skin biopsy. The biopsy is most likely
VIII-45. A 27-year-old man is seen at his primary care phy- to show:
sician’s office for evaluation of painful arthritis involving
A. Fibrinoid necrosis of small- and medium-sized
the right knee associated with diffuse bilateral finger swell-
arteries
ing. He is otherwise healthy but does recall a severe bout
B. Interface dermatitis
of diarrheal illness about 3–4 weeks prior that spontane-
C. Leukocytoclastic vasculitis with IgA deposition on
ously resolved. He works as a recreation supervisor at a
immunofluorescence
daycare center and said many of the children had a similar
D. Noncaseating granulomas
diarrheal illness. He takes no medications and reports rare
E. Septal panniculitis
marijuana use. On review of systems, he reports painful

549
VIII-50. All of the following arteries are commonly affected A. Hepatitis B surface antigen
SECTION VIII

in Takayasu arteritis EXCEPT: B. Antineutrophil cytoplasmic antibody


C. Hepatitis C polymerase chain reaction
A. Abdominal aorta
D. HIV antibody
B. Cerebral arteries
E. Rheumatoid factor
C. Common carotid artery
D. Renal arteries VIII-54. An 18-year-old man is admitted to the hospital
E. Subclavian artery with acute onset of crushing substernal chest pain that
began abruptly 30 minutes ago. He reports the pain radiat-
VIII-51. A 40-year-old male presents to the emergency
Rheumatology and Immunology

ing to his neck and right arm. He has otherwise been in


department with 2 days of low-volume hemoptysis. He
good health. He currently plays trumpet in his high school
reports that he has been coughing up 2–5 tablespoons
marching band but does not participate regularly in aero-
of blood each day. He does report mild chest pain, low-
bic activities. On physical examination, he is diaphoretic
grade fevers, and weight loss. In addition, he has had about
and tachypneic. His blood pressure is 100/48 and heart
1 year of severe upper respiratory symptoms including fre-
rate is 110 beats/min. His cardiovascular examination has
quent epistaxis and purulent discharge treated with several
a regular rhythm but is tachycardic. A 2/6 holosystolic
courses of antibiotics. Aside from mild hyperlipidemia, he
murmur is heard best at the apex and radiates to the axilla.
is otherwise healthy. His only medications are daily aspirin
His lungs have bilateral rales at the bases. The electrocardi-
and lovastatin. On physical examination he has a saddle
ogram demonstrates 4 mm of ST elevation in the anterior
nose deformity. His vital signs are normal and his lungs are
leads. On further questioning regarding his past medical
clear. A CT of the chest shows multiple 1- to 2-cm cavitat-
history, he recalls having been told that he was hospital-
ing nodules, and urinalysis shows red blood cells. Which
ized for some problem with his heart when he was 2 years
of the following tests offers the highest diagnostic yield to
old. His mother, who accompanies him, reports that he
make the appropriate diagnosis?
received aspirin and γ-globulin as treatment. Since that
A. Deep skin biopsy time, he has required intermittent follow-up with echocar-
B. Percutaneous kidney biopsy diography. What is the most likely cause of this patient’s
C. Pulmonary angiogram acute coronary syndrome?
D. Surgical lung biopsy
A. Dissection of the aortic root and left coronary ostia
E. Upper airway biopsy
B. Presence of a myocardial bridge overlying the left
VIII-52. An 84-year-old woman is seen by her primary care anterior descending artery
physician for evaluation of severe headaches. She noted C. Thrombosis of a coronary artery aneurysm
these several weeks ago, and they have been getting worse. D. Vasospasm following cocaine ingestion
Although she has not had any visual aura, she is concerned E. Vasculitis involving the left anterior descending
that she has been intermittently losing vision in her left eye artery
for the last few days. She denies new weakness or numb-
VIII-55. You are seeing in follow-up a 46-year-old man
ness, but she does report jaw pain with eating. Her past
who, 6 months ago, presented to the hospital acutely with
medical history includes coronary artery disease requiring
hemoptysis, diffuse nodular pulmonary infiltrates, and
a bypass grafting 10 years prior, diabetes mellitus, hyper-
glomerulonephritis. Workup revealed a positive serologic
lipidemia, and mild depression. Full review of symptoms
study for antibodies against cytoplasmic antineutrophil
is notable for night sweats and mild low back pain particu-
cytoplasmic antibodies (ANCA), and he was eventually
larly prominent in the morning. Which of the following is
diagnosed with granulomatosis with polyangiitis. Treat-
the next most appropriate step?
ment was initiated with high-dose glucocorticoids and
A. Aspirin 975 mg orally daily daily cyclophosphamide with excellent clinical response.
B. Measurement of erythrocyte sedimentation rate You are ready today to have the patient transition from
C. Immediate initiation of glucocorticoid treatment induction therapy with cyclophosphamide to maintenance
D. Referral for temporal artery biopsy therapy with azathioprine. What blood test should you
E. Referral for ultrasound of temporal artery check before starting azathioprine?

VIII-53. A 54-year-old man is evaluated for cutaneous vascu- A. ANCA titers


litis and peripheral nephropathy. Because of concomitant B. Cryoglobulins
renal dysfunction, he undergoes kidney biopsy that shows C. CYP3A4 genotyping
glomerulonephritis. Cryoglobulins are demonstrated in D. Glucose-6-phosphate dehydrogenase enzyme levels
the peripheral blood. Which of the following laboratory E. Thiopurine methyltransferase enzyme activity
studies should be sent to determine the etiology?

550
WWW.BOOKBAZ.IR
VIII-56. Lung biopsy has the greatest diagnostic yield in and needing to swallow multiple times for one bite of food.

SECTION VIII
which of the following vasculitic syndromes? On physical examination he has 4/5 strength in his knee
extensors, 3/5 strength in his finger flexors, and atrophy
A. Cryoglobulinemic vasculitis
in the medial thighs and forearms. On laboratory stud-
B. Cutaneous vasculitis
ies, his creatinine kinase is two times the upper limit of
C. Granulomatosis with polyangiitis (Wegener)
normal. You decide to pursue a muscle biopsy. What find-
D. IgA vasculitis (Henoch-Schönlein)
ings are likely to be noted on the muscle biopsy by light
E. Polyarteritis nodosa
microscopy?
VIII-57. All of the following conditions occur in association

QUESTIONS
A. Multifocal necrotic and regenerating muscle fibers
with Behçet syndrome EXCEPT: with a paucity of inflammation
A. Arthritis B. Perifascicular muscle atrophy
B. Deep venous thrombosis C. Perivascular inflammatory cell infiltrate with no
C. Folliculitis endomysial inflammation
D. Genital ulcers D. Rimmed vacuoles and inclusions
E. Scleritis E. Type 2 muscle fiber atrophy

VIII-58. Which of the following is required for the diagnosis VIII-61. All of the following therapeutic agents are used in
of Behçet syndrome? the treatment of polymyositis EXCEPT:

A. Large vessel vasculitis A. Azathioprine


B. Pathergy test B. Mycophenolate Mofetil
C. Recurrent oral ulceration C. Prednisone
D. Recurrent genital ulceration D. Rituximab
E. Uveitis E. Secukinumab

VIII-59. A 25-year-old woman presents with a complaint of VIII-62. A 64-year-old woman is evaluated for weakness.
painful mouth ulcerations. She describes these lesions as She has had several weeks of difficulty brushing her teeth
shallow ulcers that last for 1 or 2 weeks. The ulcers have and combing her hair. She has also noted a rash on her face.
been appearing for the last 6 months. For the last 2 days, Examination is notable for a heliotrope rash and proximal
the patient has had a painful red eye. She has had no geni- muscle weakness. Serum creatinine kinase is elevated, and
tal ulcerations, arthritis, skin rashes, or photosensitivity. she is diagnosed with dermatomyositis. After evaluation
On physical examination, the patient appears well devel- by a rheumatologist, she is found to have anti-Jo-1 anti-
oped and in no distress. She has a temperature of 37.6°C bodies. She is also likely to have which of the following
(99.7°F), heart rate of 86 beats/min, blood pressure of additional findings?
126/72, and respiratory rate of 16 breaths/min. Examina- A. Ankylosing spondylitis
tion of the oral mucosa reveals two shallow ulcers with a B. Inflammatory bowel disease
yellow base on the buccal mucosa. The ophthalmologic C. Interstitial lung disease
examination is consistent with anterior uveitis. The car- D. Primary biliary cirrhosis
diopulmonary examination is normal. She has no arthritis, E. Psoriasis
but medially on the right thigh, there is a palpable cord
in the saphenous vein. Laboratory studies reveal an eryth- VIII-63. A 63-year-old woman is evaluated for a rash on her
rocyte sedimentation rate of 68 seconds. White blood cell eyes and fatigue for 1 month. She reports difficulty with
count is 10,230/μL with a differential of 68% polymorpho- arm and leg strength and constant fatigue, but no fevers or
nuclear cells, 28% lymphocytes, and 4% monocytes. The sweats. She also has noted that she has a red discoloration
anti-nuclear antibody and anti-dsDNA antibody are nega- around her eyes. She has hypothyroidism but is otherwise
tive, and C3 is 89 mg/dL and C4 is 24 mg/dL. What is the well. On examination, she has a heliotrope rash and proxi-
most likely diagnosis? mal muscle weakness. A diagnosis of dermatomyositis is
made after demonstration of elevated serum creatinine
A. Behçet syndrome
kinase and confirmatory electromyograms. Which of the
B. Bullous pemphigoid
following studies should be performed as well to look for
C. Discoid lupus erythematosus
associated conditions?
D. Sjögren syndrome
E. Systemic lupus erythematosus A. Mammogram
B. Serum anti-nuclear antibody measurement
VIII-60. A 58-year-old man presents to the clinic for a C. Stool examination for ova and parasites
routine follow-up visit. He notes he has been dropping D. Thyroid-stimulating immunoglobulins
things more frequently and is having trouble rising from E. Titers of antibodies to varicella zoster
a seated position. He also notes some trouble swallowing

551
VIII-64. You are seeing your long-term patient in the clinic.
SECTION VIII

He has a history of coronary artery disease, with a lateral


myocardial infarction 1 year ago. At that time, he was
started on simvastatin, aspirin, metoprolol, and lisinopril.
About 2 months ago, he started noting thigh and shoul-
der soreness. One month after onset, his muscle pain had
increased and he was noticing weakness. His creatinine
kinase (CK) was elevated to eight times the upper limit
of normal. His simvastatin was discontinued 3 weeks ago.
Rheumatology and Immunology

Today, he reports that his pain has continued and, if any-


thing, is worse than a month ago. His CK is 12 times the
upper limit of normal. What is the next best test to estab-
lish a diagnosis?
A. Antibody against 3-hydroxy-3-methylglutaryl–coen-
zyme A reductase
B. Anti-nuclear antibody
C. Anti-Jo-1 antibody
D. Antibody against signal recognition particle
E. Aldolase levels
FIGURE VIII-68
VIII-65. Which of the following disorders is associated with
relapsing polychondritis? bilateral hilar lymphadenopathy. What type of skin rash is
present?
A. Giant cell arteritis
B. Granulomatosis with polyangiitis (formerly Wegener A. Acne vulgaris
disease) B. Acute cutaneous lupus
C. Rheumatoid arthritis C. Erythema nodosum
D. Psoriasis D. Heliotrope rash
E. B and C E. Lupus pernio

VIII-66. A 47-year-old man is evaluated for 1 year of recur- VIII-69. All of the following have been implicated in the
rent episodes of bilateral ear swelling. The ear is painful proposed pathogenesis of sarcoidosis EXCEPT:
during these events, and the right ear has become floppy. A. Exposure to mold
He is otherwise healthy and reports no illicit habits. He B. Genetic susceptibility
works in an office, and his only sport is tennis. On exami- C. Immune response to mycobacterial proteins
nation, the left ear has a beefy red color and the pinna is D. Infection with Propionibacterium acnes
tender and swollen; the earlobe appears minimally swollen E. Malignant expansion of helper T cells
but is neither red nor tender. Which of the following is the
most likely explanation for this finding? VIII-70. Which of the following statements regarding pul-
monary sarcoidosis is true?
A. Behçet syndrome
B. Cogan syndrome A. Lung involvement is the second most common
C. Hemoglobinopathy manifestation of sarcoidosis, behind only cutaneous
D. Recurrent trauma involvement.
E. Relapsing polychondritis B. Obstructive disease is a rare manifestation of pulmo-
nary sarcoidosis.
VIII-67. Which of the following cardiovascular conditions C. Pulmonary hypertension never responds to therapy
can occur in sarcoidosis as a result of granulomatous in sarcoidosis patients.
infiltration? D. Pulmonary infiltrates in sarcoidosis tend to be pre-
A. Abdominal aortic aneurysm dominantly an upper lobe process.
B. Coronary artery disease E. The presence of cough should prompt evaluation for
C. Decreased ejection fraction a cause other than pulmonary sarcoidosis.
D. Ventricular arrhythmia VIII-71. You are seeing a 55-year-old white man with a
E. C and D history of sarcoidosis. He ran out of prednisone about
VIII-68. A 40-year-old African American woman presents to 2 months prior to seeing you and, except for some con-
the clinic with a facial rash. The rash has lasted for several stipation, feels well. A metabolic panel reveals a calcium
months and is raised and erythematous (Figure VIII-68). of 12.2 mg/dL (normal up to 10.5 mg/dL). You know that
She also reports a new dry cough. Chest radiograph shows sarcoidosis can be associated with hypercalcemia. Which

552
WWW.BOOKBAZ.IR
of the following is the correct mechanism for sarcoidosis- with periodic fever syndrome, and tumor necrosis factor-

SECTION VIII
associated hypercalcemia? receptor-associated periodic syndrome?
A. Direct granulomatous involvement of the axial skel- A. Adalimumab
eton causing calcium release from bones B. Canakinumab
B. Direct stimulation of increased intestinal calcium C. Colchicine
absorption D. Lisinopril
C. Increased parathyroid hormone production E. Rituximab
D. Increased production of 1,25-dihydroxyvitamin D
VIII-75. You are seeing a 19-year-old woman today in con-

QUESTIONS
E. Increased production of 25-hydroxyvitamin D
sultation for recurrent fevers. She reports several years
VIII-72. All of the following conditions are manifestations of fevers, occurring on average every 2–3 months. These
of IgG4-related disease EXCEPT: episodes are unpredictable, although she thinks they may
occur in times of psychological stress. Each febrile episode
A. Autoimmune pancreatitis
lasts 2–3 days. She also has recurrent episodes of abdomi-
B. Crescentic glomerulonephritis
nal pain. Repeated blood cultures have been negative,
C. Lymphoplasmacytic aortitis
even during acute febrile episodes. Similarly, abdominal
D. Orbital pseudotumor
CT scans have shown no obvious etiology for her pain.
E. Sialadenitis
During one episode, she underwent an exploratory lapa-
VIII-73. Your patient is a 34-year-old man who presented rotomy, which showed peritoneal adhesions and a sterile
to you with unexplained pancreatitis 2 weeks ago. Imag- neutrophilic peritoneal exudate. She also notes that when
ing of his pancreas showed diffuse pancreatic enlarge- she exercises, she develops intense muscle pains that last
ment. He denies any alcohol intake and did not have any for days. An extensive serologic search for autoantibod-
gallstones on imaging. Interestingly, on examination, he ies returned negative, including anti-nuclear antibodies.
also has marked lacrimal gland and submandibular gland Which of the following is the most likely diagnosis?
enlargement. Biopsy of his submandibular gland is pic- A. Familial Mediterranean fever
tured in Figure VIII-73. The cells pictured in the figure B. Lymphoma
stain strongly positive for IgG4, CD19, and CD138. Which C. Relapsing fever
of the following is the appropriate therapy? D. Subacute bacterial endocarditis
E. Systemic lupus erythematosus

VIII-76. For the patient described in question VIII-75,


which of the following medications would you prescribe
to reduce attacks and help prevent the development of sys-
temic amyloidosis?
A. Colchicine
B. Cyclosporine
C. Prednisone
D. Rituximab
E. Vancomycin

VIII-77. In the United States, what percentage of patients


over the age of 60 years have symptomatic osteoarthritis
of the knee?
A. 1%
FIGURE VIII-73 B. 5%
C. 12%
D. 30%
A. Thalidomide and dexamethasone E. 43%
B. Cytomegalovirus immunoglobulin and ganciclovir
C. Systemic chemotherapy VIII-78. Which of the following joints is often spared by
D. Prednisone osteoarthritis?
E. Anakinra
A. Cervical spine
VIII-74. Which of the following drugs can be used to B. Distal interphalangeal joint
treat several genetic periodic fever syndromes including C. Hip
Muckle-Wells syndrome, hyperimmunoglobulinemia D D. Proximal interphalangeal joint
E. Wrist

553
VIII-79. Which of the following statements regarding osteo-
SECTION VIII

arthritis is true?
A. During the diagnostic workup of a suspected osteo-
arthritic joint, MRI is warranted to evaluate for any
other causes.
B. Loss of cartilage causes pain due to direct stimulation
of pain receptors in joint cartilage itself.
C. Osteoarthritis is the second most common cause of
Rheumatology and Immunology

arthritis, behind rheumatoid arthritis.


D. Synovial fluid white blood cell count is usually
<1000 cells/μL in osteoarthritis.
E. The severity of radiographic changes in osteoarthritis
correlates well with symptoms.
FIGURE VIII-82A
VIII-80. You are seeing a 60-year-old obese woman with
bilateral knee osteoarthritis. She describes pain most days
and limiting pain at least 2 days per week. She has tried VIII-83. You are planning on starting allopurinol for a
activity modification (walking less) without success. All 55-year-old woman for a new diagnosis of gouty arthri-
of the following therapies have been shown to be effica- tis. Which of the following statements best describes an
cious in treating osteoarthritis symptoms in this patient appropriate dosing strategy for allopurinol?
EXCEPT:
A. Allopurinol and azathioprine are commonly used
A. Acetaminophen together in the treatment of gout.
B. Glucocorticoid intra-articular injections B. Allopurinol dosing should be adjusted for liver
C. Glucosamine-chondroitin function.
D. Naproxen C. Allopurinol dosing should be titrated to achieve a
E. Total joint arthroplasty serum uric acid level <6 mg/dL.
D. Allopurinol should be avoided when patients are tak-
VIII-81. A 75-year-old woman presents to the clinic with
ing colchicine.
a newly swollen right knee. She denies any fevers, night
E. Allopurinol toxicity is more common in patients
sweats, or other constitutional symptoms. She does not
expressing human leukocyte antigen-B27.
have any toe, foot, or ankle pain or swelling. She has never
had a similar episode previously. She has a history of VIII-84. Which of the following findings is typically seen in
osteoarthritis and underwent a left total hip arthroplasty patients with fibromyalgia?
5 years ago. On examination, her right knee is warm with
a moderate size effusion and tender to palpation. Labora- A. Elevated C-reactive protein
tory studies show a uric acid level of 5.2 mg/dL (normal). B. Elevated thyroid-stimulating hormone
A radiograph of the knee shows a linear radiodensity in C. Erosions of the metacarpophalangeal joints on
the joint space consistent with chondrocalcinosis. What is radiographs
the most likely diagnosis? D. Positive anti-nuclear antibody
E. Skin roll tenderness on physical examination
A. Calcium pyrophosphate arthritis (pseudogout)
B. Gout VIII-85. A 42-year-old woman is seen in her primary care
C. Rheumatoid arthritis doctor’s office complaining of diffuse pains and fatigue.
D. Septic arthritis She has a difficult time localizing the pain to any particu-
lar joint or location, but she reports it affects her upper
VIII-82. Mr. Hinsley is a 72-year-old man with only a his- and lower extremities, neck, and hips. It is described as
tory of hypertension on hydrochlorothiazide. He presents achy and 10 out of 10 in intensity. She feels that her joints
today with acute, excruciating knee pain. On examination, are stiff but does not notice that it is worse in the morn-
his knee is warm, mildly erythematous, swollen, and ten- ing. The pain has been present for the last 6 months and
der to the touch or passive movement. Microscopic exami- is increasing in intensity. She has tried both over-the-
nation of joint fluid is shown in Figure VIII-82A. Which counter ibuprofen and acetaminophen without significant
of the following is Mr. Hinsley’s most likely metabolic relief. The patient feels as if the pain is interfering with
derangement? her ability to get restful sleep and is making it difficult for
A. Acute bacterial joint infection her to concentrate. She has missed multiple days of work
B. Antibodies to anti-nuclear antigens as a waitress and fears that she will lose her job. There is
C. Hyaline cartilage degeneration a medical history of depression and obesity. The patient
D. Increased production of inorganic pyrophosphate is currently taking venlafaxine sustained release 150 mg
E. Uric acid overproduction daily. She has a family history of rheumatoid arthritis
in her mother. She smokes one pack of cigarettes daily.

554
WWW.BOOKBAZ.IR
On physical examination, vital signs are normal. Body VIII-88. A 42-year-old man is found to have the finding

SECTION VIII
mass index is 36 kg/m2. Joint examination demonstrates seen in Figure VIII-88 on a physical examination. All of
no erythema, swelling, or effusions. There is diffuse pain the following conditions are associated with this finding
with palpation at the insertion points of the suboccipital EXCEPT:
muscles, at the midpoint of the upper border of the trape-
zius muscle, along the second costochondral junction, at
the lateral epicondyles, and along the medial fat pad of the
knees. All of the following statements regarding the cause
of this patient’s diffuse pain syndrome are true EXCEPT:

QUESTIONS
A. Cognitive dysfunction, sleep disturbance, anxiety,
and depression are common comorbid neuropsycho-
logical conditions.
B. Pain in this syndrome is associated with increased
evoked pain sensitivity.
C. Pain in this syndrome is often localized to specific
joints.
D. This syndrome is present in 2–5% of the general pop-
ulation, but increases in prevalence to 20% or more FIGURE VIII-88 Reproduced with permission from Kang S et al:
of patients with degenerative or inflammatory rheu- Fitzpatrick’s Dermatology, 9th ed. New York: McGraw Hill, 2019.
matic disorders.
E. Women are nine times more likely than men to be
affected by this syndrome. A. Chronic obstructive pulmonary disease
B. Cyanotic congenital heart disease
VIII-86. A 36-year-old woman presents to your office C. Cystic fibrosis
with diffuse pain throughout her body associated with D. Hepatocellular carcinoma
fatigue, insomnia, and difficulty concentrating. She finds E. Hyperthyroidism
the pain difficult to localize but reports that it is 7–8 out
of 10 in intensity and not relieved by nonsteroidal anti- VIII-89. A 52-year-old man presented to his primary care
inflammatory medications. She has a long-standing his- physician complaining of new-onset pain in the knuckles
tory of generalized anxiety disorder and is treated with of his index and middle fingers of both hands. On exami-
sertraline 100 mg daily as well as clonazepam 1 mg twice nation, the second and third metacarpophalangeal (MCP)
daily. On examination, she has pain with palpation at sev- joints of both hands are swollen and tender. The rest of his
eral musculoskeletal sites. Her laboratory examination physical examination is normal. His past medical history
demonstrates a normal complete blood count, basic meta- is only notable for hyperlipidemia controlled with atorv-
bolic panel, erythrocyte sedimentation rate, and rheuma- astatin. His laboratory studies are notable for an elevated
toid factor. You diagnose her with fibromyalgia. All of the ferritin, and after demonstration of a mutation of the HFE
following therapies are recommended as part of the treat- gene, he is diagnosed with hemochromatosis. Which of
ment plan for fibromyalgia EXCEPT: the following statements regarding his joint abnormalities
is true?
A. An exercise program that includes strength training,
aerobic exercise, and yoga A. The second and third finger MCPs are also typically
B. Cognitive-behavioral therapy for insomnia involved in osteoarthritis.
C. Milnacipran B. Arthropathy is unlikely related to hemochromatosis.
D. Oxycodone C. Arthropathy may progress with phlebotomy.
E. Pregabalin D. Arthropathy occurs in less than 20% of patients with
hemochromatosis.
VIII-87. Which of the following conditions is the most fre- E. Radiographs are likely to show erosions in the MCPs.
quent cause of neuropathic joint disease (Charcot joint)?
VIII-90. All of the following can cause bursitis EXCEPT:
A. Amyloidosis
B. Congenital indifference to pain A. Bacterial infection
C. Diabetes mellitus B. Fibromyalgia
D. Syringomyelia C. Gout
E. Tabes dorsalis D. Rheumatoid arthritis
E. Overuse

555
VIII-91. A 57-year-old man comes to the clinic with new The patient’s past medical history is also significant for
SECTION VIII

pain in his right elbow. He works for an international diabetes mellitus, for which she takes metformin and gly-
corporation and travels frequently for work. This travel buride. On physical examination, the right shoulder is
requires him to carry and pull suitcases through the air- not warm or red but is tender to touch. Passive and active
port on his weekly trips out of town. He notes that the pain range of motion are limited in flexion, extension, and
is worse when shaking hands at business meetings. On abduction. A right shoulder radiogram shows osteopenia
examination, no swelling is noted in the elbow or other without evidence of joint erosion or osteophytes. What is
joints. The pain is reproducible by palpation of the lateral the most likely diagnosis?
elbow. Which of the following is an option for treatment of
Rheumatology and Immunology

A. Adhesive capsulitis
his musculoskeletal condition?
B. Avascular necrosis
A. Adalimumab C. Bicipital tendinitis
B. Corticosteroid injection at the lateral epicondyle D. Osteoarthritis
C. Methotrexate E. Rotator cuff tear
D. Nonsteroidal anti-inflammatory drugs and rest
E. B and D VIII-94. A 32-year-old woman presents to the clinic with
right thumb and wrist pain that has worsened over several
VIII-92. A 32-year-old woman is seen in the clinic with a weeks. She has pain when she pinches her thumb against
complaint of left knee pain. She enjoys running long dis- her other fingers. Her only other history is that she is a
tances and is currently training for a marathon. She is new mother with an 8-week-old infant at home. On physi-
running on average 30–40 miles weekly. She currently is cal examination, she has mild swelling and tenderness
experiencing an aching pain on the lateral aspect of her over the radial styloid process, and pain is elicited when
left knee. There is a burning sensation that also continues she places her thumb in her palm and grasps it with her
up the lateral aspect of her thigh. She denies any injury to fingers. A Phalen maneuver is negative. Which condition
her knee, and she has not felt that it was hot or swollen. She is most likely?
is otherwise healthy and takes no medications other than
A. Carpal tunnel syndrome
herbal supplements. Physical examination of the knee
B. De Quervain tenosynovitis
reveals point tenderness over the lateral femoral condyle
C. Gouty arthritis of the first metacarpophalangeal joint
that is worse with flexing the knee. The patient is asked to
D. Palmar fasciitis
lie on her right side with her right knee and hip flexed at
E. Rheumatoid arthritis
90 degrees. Her left leg is extended at the hip and slowly
lowered into adduction behind the bottom leg, reproduc- VIII-95. You are evaluating a 42-year-old woman who com-
ing the patient’s left knee pain. All of the following treat- plains of pain on the underside of her right heel that is
ments can be recommended for this patient EXCEPT: excruciating in the morning when she first walks from bed
A. Assessment of the patient’s running shoes to ensure a to the bathroom. The pain improves somewhat during the
proper fit morning but again worsens mid-day particularly when
B. Glucocorticoid injection so as not to interfere with climbing stairs. She has a past medical history of hyper-
the patient’s continued preparation for the upcoming tension, smokes one pack per day of cigarettes, and works
marathon as a waitress at a diner. Medications include hydrochlo-
C. Ibuprofen 600–800 mg every 6 hours as needed for rothiazide and oral contraceptives. Physical examination
pain is unremarkable except for flat feet and focal tenderness
D. Referral for physical therapy on the bottom of the right heel. There is no tenderness at
E. Referral for surgical release if conservative therapy the ankle or calf, and the diameters of the lower legs are
fails equivalent. A radiograph of the right heel and ankle shows
only heel spurs. All of the following statements regarding
VIII-93. A 58-year-old woman presents complaining of her condition are true EXCEPT:
right shoulder pain. She does not recall any prior injury
A. Heel spurs are not diagnostic.
but notes that she feels that the shoulder has been get-
B. Local glucocorticoid injection incurs a risk of plantar
ting progressively stiffer over the last several months. She
fascia rupture.
previously had several episodes of bursitis of the right
C. Oral contraceptives and smoking are risk factors.
shoulder that were treated successfully with nonsteroidal
D. Orthotic shoe implants may be beneficial.
anti-inflammatory drugs and corticosteroid injections.
E. The prognosis for improvement is good.

556
WWW.BOOKBAZ.IR
SECTION VI
Disorders of the Kidney and Urinary Tract

QUESTIONS

DIRECTIONS: Choose the one best response to each question. VI-4. Which of the following procedures has the highest risk
for postoperative acute kidney injury?
A. Anterior cruciate ligament repair
VI-1. Which segment of the kidney reabsorbs the highest B. Cataract removal
percentage of filtered sodium chloride? C. Coronary artery bypass
D. Thyroidectomy
A. Collecting duct
E. Total hip arthroplasty
B. Distal convoluted tubule
C. Loop of Henle VI-5. Which of the following is a potential etiology for
D. Proximal convoluted tubule ischemic acute renal failure?
VI-2. A 33-year-old woman with recently treated acute A. Apoptosis and necrosis of tubular cells
myelogenous leukemia now in remission is admitted to B. Decreased glomerular vasodilation in response to
the hospital with lethargy, fever, and tachycardia. Blood nitric oxide
cultures grow Pseudomonas that is resistant to cefepime. C. Increased glomerular vasoconstriction in response to
She is started on IV gentamicin. Five days after starting elevated endothelin levels
gentamicin, her serum creatinine rises from her baseline D. Increased leukocyte adhesion within the glomerulus
of 1.0 mg/dL to 2.4 mg/dL. No red or white cell casts are E. All of the above
seen on her urinalysis. Her magnesium level is decreased
at 1.5 mg/dL. Renal ultrasound is unremarkable with no VI-6. A 57-year-old man with a history of diabetes mellitus
hydronephrosis. Which of the following is the most likely and chronic kidney disease with a baseline creatinine of
mechanism of her acute kidney injury? 1.8 mg/dL underwent cardiac catheterization for acute
myocardial infarction. He is subsequently diagnosed with
A. Acute interstitial nephritis acute kidney injury related to iodinated contrast. All of the
B. Acute tubular necrosis following statements are true regarding his acute kidney
C. Glomerulonephritis injury EXCEPT:
D. Ischemic injury
E. Obstruction A. Fractional excretion of sodium will be low.
B. His creatinine is likely to peak within 3–5 days.
VI-3. Which of the following laboratory abnormalities typi- C. His diabetes mellitus predisposed him to develop
cally can be seen accompanying acute kidney injury? contrast nephropathy.
D. Transient tubule obstruction with precipitated iodi-
A. Hypercalcemia
nated contrast contributed to development of his
B. Hypokalemia
acute kidney injury.
C. Hyponatremia
E. White blood cell casts are likely on microscopic
D. Hypophosphatemia
examination of urinary sediment.
E. Metabolic alkalosis

457
VI-7. Which of the following acute kidney injury patients is of 55 mL/min/1.73 m2. His hemoglobin A1c is 5.4%. He
most likely to have evidence of bilateral hydronephrosis on is currently on metformin, naproxen, and hydrochlorothi-
SECTION VI

ultrasound evaluation of the kidneys? azide. What medication change could help slow his pro-
gression of chronic kidney disease?
A. A 19-year-old man with purpura fulminans associ-
ated with gonococcal sepsis A. Add furosemide
B. A 37-year-old woman undergoing chemotherapy B. Discontinue metformin
and radiation for advanced cervical cancer C. Discontinue naproxen
C. A 48-year-old man with chronic renal insufficiency D. Substitute lisinopril for hydrochlorothiazide
Disorders of the Kidney and Urinary Tract

due to hypertension that received iodinated contrast E. C and D


for an abdominal angiogram
D. A 53-year-old man with Escherichia coli 0157:H7– VI-12. Which of the following patients has the greatest risk
associated thrombotic thrombocytopenic purpura of progression to chronic kidney disease?
E. An 85-year-old woman who resides in a nursing A. A 30-year-old man with an estimated glomeru-
home with pyelonephritis and sepsis lar filtration rate (GFR) of 50 mL/min/1.73 m2 and
350 mg/g of persistent albuminuria
VI-8. A 54-year-old man is admitted to the medical inten-
B. A 45-year-old man with an estimated GFR of 90 mL/
sive care unit with sepsis associated with pneumococcal
min/1.73 m2 and <30 mg/g of persistent albuminuria
pneumonia. He requires mechanical ventilation as well
C. A 55-year-old man with an estimated GFR of 70 mL/
as norepinephrine to maintain a mean arterial pressure
min/1.73 m2 and 100 mg/g of persistent albuminuria
>60 mmHg. Invasive hemodynamics show adequate left
D. A 65-year-old woman with an estimated GFR of
heart filling pressures, and he is not known to have left
65 mL/ min/1.73 m2 and <30 mg/g of persistent
ventricular dysfunction. On the third hospital day, his
albuminuria
urine output drops and creatinine increases to 3.4 mg/dL.
E. A 75-year-old man with an estimated GFR of 35 mL/
Acute tubular injury is diagnosed. Which of the following
min/1.73 m2 and <30 mg/g of persistent albuminuria
agents has been shown to improve outcomes associated
with his acute tubular injury? VI-13. In stage V chronic kidney disease, glomerular filtra-
A. Furosemide tion rate is below which of the following levels?
B. Bosentan A. 90 mL/min/1.73 m2
C. Low-dose dopamine B. 60 mL/min/1.73 m2
D. Insulin-like growth factor C. 25 mL/min/1.73 m2
E. None of the above D. 15 mL/min/1.73 m2
E. 0 mL/min/1.73 m2 (anuria)
VI-9. It is hospital day 5 for a 65-year-old patient with pre-
renal azotemia secondary to dehydration. His creatinine VI-14. All of the following statements regarding the use of
was initially 3.6 mg/dL on admission, but it has improved exogenous erythropoietin in patients with chronic kidney
today to 2.1 mg/dL. He complains of mild lower back pain, disease are true EXCEPT:
and you prescribe naproxen to be taken intermittently. By
what mechanism might this drug further impair his renal A. Exogenous erythropoietin should be adminis-
function? tered with a target hemoglobin concentration of
100–115 g/L.
A. Afferent arteriolar vasoconstriction B. Use of exogenous erythropoietin is associated with
B. Afferent arteriolar vasodilatation improved cardiovascular outcomes.
C. Efferent arteriolar vasoconstriction C. Use of exogenous erythropoietin is associated with
D. Proximal tubular toxicity increased risk of stroke in patients with concomitant
E. Ureteral obstruction type 2 diabetes mellitus.
D. Use of exogenous erythropoietin may be associated
VI-10. Which of the following is the leading cause of chronic
with faster progression to the need for dialysis.
kidney disease in North America?
E. Use of exogenous erythropoietin is associated with
A. Autosomal dominant polycystic kidney disease increased incidence of thromboembolic events.
B. Contrast nephropathy
C. Diabetic nephropathy VI-15. A 63-year-old woman with chronic kidney disease is
D. Focal segmental glomerulosclerosis maintained on daily peritoneal dialysis. Her past medical
E. Recurrent renal calculi history is notable for hypertension and atrial fibrillation.
Her medications include losartan and warfarin. Two days
VI-11. A 65-year-old man with a history of hypertension, ago, she noticed a small painful nodule on her abdomen
diabetes, and chronic low back pain presents to the clinic that has progressed to involve skin necrosis and ulceration
for follow-up. His physical examination is unremarkable of the abdominal wall (Figure VI-15). All of the following
with no edema or jugular venous distension. Recent labo- statements regarding her condition are true EXCEPT:
ratory testing shows a decreased glomerular filtration rate

458
WWW.BOOKBAZ.IR
VI-19. A 35-year-old woman with hypertensive kidney dis-
ease progresses to end-stage renal disease. She was initi-

SECTION VI
ated on peritoneal dialysis 1 year ago and has done well
with relief of her uremic symptoms. She is brought to the
emergency department with fever, altered mental status,
diffuse abdominal pain, and cloudy dialysate. Her peri-
toneal fluid is withdrawn through her catheter and sent
to the laboratory for analysis. The fluid white blood cell
count is 125/μL with 85% polymorphonuclear neutro-

QUESTIONS
phils. Which organism is most likely to be found in the
culture of the peritoneal fluid?
A. Candida albicans
B. Escherichia coli
C. Mycobacterium tuberculosis
D. Pseudomonas aeruginosa
FIGURE VI-15 E. Staphylococcus epidermidis

VI-20. A 45-year-old woman begins hemodialysis for end-


A. Oral calcium supplement may be a risk factor. stage renal disease associated with diabetes mellitus.
B. Pathologically, there is vascular occlusion. Which of the following is her most likely eventual cause
C. Pseudomonas co-infection is typical. of death?
D. Severe hyperparathyroidism may not be present. A. Dementia
E. Warfarin is a risk factor for development of the lesion. B. Major bleeding episode
C. Myocardial infarction
VI-16. Which of the following is the most common acute
D. Progressive uremia
complication of hemodialysis?
E. Sepsis
A. Anaphylactoid reactions to dialyzer
B. Bleeding from access site VI-21. Your patient with end-stage renal disease on hemodi-
C. Hypertension alysis has persistent hyperkalemia. He has a history of total
D. Hypotension bilateral renal artery stenosis, which is why he is on hemo-
E. Muscle cramps dialysis. He only has electrocardiogram changes when his
potassium rises above 6.0 mEq/L, which occurs a few times
VI-17. A 60-year-old woman with hypertension and type 2 per week. You admit him to the hospital for further evalu-
diabetes has recently started peritoneal dialysis due to ation. Your laboratory evaluation, nutrition counseling,
chronic kidney disease from diabetic nephropathy. Which and medication adjustments have not impacted his serum
complications of peritoneal dialysis are of concern for this potassium. What is the next reasonable step to undertake
patient? for this patient?
A. Hyperkalemia A. Adjust the dialysate
B. Hyperglycemia B. Administer a daily dose of furosemide
C. Hyperphosphatemia C. Perform “sodium modeling”
D. Hypertriglyceridemia D. Implant an automatic defibrillator
E. B and D E. Perform bilateral nephrectomy

VI-18. A patient is followed closely by her nephrologist for VI-22. Which of the following medications used for immu-
stage IV chronic kidney disease associated with focal seg- nosuppression after kidney transplant works by inhibiting
mental glomerulosclerosis. Which of the following is an purine synthesis and can cause diarrhea as a common side
indication for initiation of maintenance hemodialysis? effect?
A. Acidosis controlled with daily bicarbonate A. Belatacept
administration B. Cyclosporine
B. Bleeding diathesis C. Mycophenolate mofetil
C. Blood urea nitrogen >110 mg/dL without symptoms D. Sirolimus
D. Creatinine >5 mg/dL without symptoms E. Tacrolimus
E. Hyperkalemia controlled with sodium polystyrene

459
VI-23. All of the following are considered expanded donor VI-28. A 50-year-old obese female with a 5-year history
criteria for renal transplantation EXCEPT: of mild hypertension controlled by a thiazide diuretic is
SECTION VI

being evaluated because proteinuria was noted on a urine


A. Deceased donor >60 years
dipstick during her routine yearly medical visit. Physical
B. Deceased donor >50 years, hypertension, and creati-
examination discloses a height of 167.6 cm (66 inches),
nine >1.5 mg/dL
weight of 91 kg (202 lb), blood pressure of 130/80, ele-
C. Deceased donor >50 years, hypertension, and death
vated jugular venous pressure, a fourth heart sound,
caused by cerebrovascular accident (CVA)
and trace pedal edema. Laboratory values are as follows:
D. Deceased donor >50 years, death caused by CVA,
serum creatinine 106 μmol/L (1.2 mg/dL), blood urea
Disorders of the Kidney and Urinary Tract

and creatinine >1.5 mg/dL


nitrogen 6.4 mmol/L (18 mg/dL), and creatinine clear-
E. Presence of antibodies against the donor kid-
ance 87 mL/min. Urinalysis shows pH 5.0, specific grav-
ney in the recipient at the time of the anticipated
ity 1.018, protein 3+, no glucose, and occasional coarse
transplantation
granular cast. Urine protein excretion is 5.9 g/dL. A renal
VI-24. Which of the following clinical features is character- biopsy demonstrates that 60% of the glomeruli, mostly in
istic of patients with nephropathy due to type 1 diabetes? the corticomedullary junction, have segmental scarring
by light microscopy, with the remainder of the glomeruli
A. Kidney size is reduced appearing unremarkable (Figure VI-28). Which of the fol-
B. Most patients also have diabetic retinopathy lowing is the most likely diagnosis?
C. Proteinuria is uncommon
D. Renal failure occurs within 1–2 years after the onset
of proteinuria
E. Urinalysis shows red blood cell casts

VI-25. All of the following are typical causes of secondary


membranous glomerulonephritis EXCEPT?
A. Hepatitis C
B. Malignancy
C. Nonsteroidal anti-inflammatory drug use
D. Systemic lupus erythematosus
E. Tuberculosis

VI-26. A 33-year-old previously healthy woman presents to


the emergency department with malaise, confusion, and
fever of 37.8°C (100.1°F). She had no preceding diarrheal
illness. CT of the head is unremarkable. She is found to have
platelets of 13,000/cu, hemoglobin of 9.0 g/dL, and creati- FIGURE VI-28 EGN/UPenn Collection.
nine of 2.3 mg/dL. Urinalysis shows pH 5.0, protein 1+,
and 11 red blood cells/high-powered field. Which of the A. Focal segmental sclerosis
following laboratory findings would be expected with her B. Hypertensive nephrosclerosis
diagnosis? C. Minimal change (nil) disease
D. Membranous glomerulopathy
A. ADAMSTS13 <10% E. Crescentic glomerulonephritis
B. Anti-glomerular basement membrane antibodies
C. Decreased lactate dehydrogenase VI-29. Which of the following is a genetic disease that can
D. Positive direct Coombs test lead to bilateral renal cysts and carries an increased risk of
E. Spherocytes on peripheral blood smear renal cell carcinoma?

VI-27. A 21-year-old man is diagnosed with poststreptococ- A. Medullary sponge kidney


cal glomerulonephritis. Which of the following is likely to B. Neurofibromatosis type 1
be found in his urine? C. Tuberous sclerosis
D. Von Hippel-Lindau disease
A. >3 g per 24-hour proteinuria without hematuria E. C and D
B. Macroscopic hematuria and 24-hour urinary albu-
min of 227 mg VI-30. A 28-year-old woman was recently diagnosed with
C. Microscopic hematuria with leukocytes and 24-hour autosomal dominant polycystic kidney disease after an
urinary albumin of 227 mg episode of hematuria. She is concerned about intracranial
D. Positive urine culture for Streptococcus aneurysm risk. Which of the following statements is true
E. Sterile pyuria without proteinuria regarding this risk?

460
WWW.BOOKBAZ.IR
A. Family history of ruptured intracranial aneurysms of 15 mEq/L, potassium of 10 mEq/L, and chloride of
does not increase risk of rupture. 12 mEq/L. What is the most likely diagnosis?

SECTION VI
B. Prior intracranial hemorrhage does not increase risk
A. Chronic diarrhea
of subsequent hemorrhage.
B. Type I renal tubular acidosis (RTA)
C. The size of the aneurysm does not correlate with its
C. Type II RTA
risk of spontaneous rupture.
D. Type III RTA
D. There is no increased risk of intracranial aneurysm
E. Type IV RTA
in this condition.
E. Uncontrolled hypertension augments the risk of VI-35. In which of the following cases would treatment with

QUESTIONS
spontaneous rupture. corticosteroids for biopsy-proven interstitial nephritis be
most likely to impact long-term renal recovery?
VI-31. A 21-year-old male college student is evaluated for
profound fatigue that has been present for several years but A. A 37-year-old woman with sarcoidosis
has recently become debilitating. He also reports several B. A 48-year-old man with slowly progressing intersti-
foot spasms and cramps and occasionally sustained mus- tial nephritis over 2 months with fibrosis found on
cle contractions that are uncontrollable. He is otherwise biopsy
healthy, takes no medications, and denies tobacco or alco- C. A 54-year-old man with diabetes mellitus and recent
hol use. On examination, he is well developed with nor- Salmonella infection
mal vital signs including blood pressure. The remainder of D. A 63-year-old man with allergic interstitial nephritis
the examination is normal. Laboratory evaluation shows a after cephalosporin antibiotic use
sodium of 138 mEq/L, potassium of 2.8 mEq/L, chloride E. None of the above
of 90 mEq/L, and bicarbonate of 30 mmol/L. Magnesium
level is normal. Urine screen for diuretics is negative, and VI-36. A 58-year-old woman undergoes a hysterectomy
urine chloride is elevated. Which of the following is the and postoperatively develops acute respiratory distress
most likely diagnosis? syndrome. She is treated with mechanical ventilation and
broad-spectrum antibiotics. Aside from hypothyroidism,
A. Bulimia nervosa she has no underlying medical conditions. On day 5 of
B. Diuretic abuse her hospitalization, her urine output is noted to fall, and
C. Gitelman syndrome her serum creatinine rises from 1.2 to 2.5 mg/dL. Allergic
D. Liddle syndrome interstitial nephritis from cephalosporin antibiotics is sus-
E. Type 1 pseudohypoaldosteronism pected. Which of the following findings will confirm this
diagnosis?
VI-32. A 24-year-old Caucasian man presents to the clinic
after being found to have a creatinine of 1.6 mg/dL on A. Hematuria
routine laboratory studies. The rest of his metabolic panel B. Peripheral blood eosinophilia
and blood counts are normal. He has been mostly healthy, C. Urinary eosinophils on urine microscopy
with the exception of several urinary tract infections in his D. White blood cell casts on urine microscopy
childhood. He takes no medications. Urinalysis shows 1+ E. None of the above
proteinuria and no red or white blood cells. Renal ultra-
sound shows decreased size of both kidneys with thinned VI-37. Eculizumab, used in the treatment of atypical hemo-
cortices and regions of compensatory hypertrophy. What lytic uremic syndrome, targets which of the following?
is the likely cause of his renal disease? A. C5
A. Allergic interstitial nephritis B. Factor V
B. IgG4 disease C. Interleukin (IL)-6 receptor
C. Reflux nephropathy D. IL-17
D. Sickle cell nephropathy E. Tumor necrosis factor-α
E. Sjögren syndrome
VI-38. A 66-year-old woman presents to the emergency
VI-33. All of the following medications can cause acute room with malaise, confusion, and headache. Blood
interstitial nephritis EXCEPT: pressure is found to be 220/105. Physical examination is
notable for skin thickening of the arms and legs, facial
A. Celecoxib telangiectasias, and decreased oral aperture. Laboratory
B. Hydromorphone studies show a creatinine of 3.5 mg/dL, hemoglobin of
C. Pantoprazole 7.0 g/dL, platelets of 75,000/dL, lactate dehydrogenase of
D. Penicillin 700. What is the appropriate initial treatment?
E. Valproate
A. Amlodipine
VI-34. A patient with a history of Sjögren syndrome has the B. Captopril
following laboratory findings: plasma sodium 139 mEq/L, C. Eculizumab
chloride 112 mEq/L, bicarbonate 15 mEq/L, and potas- D. Heparin
sium 3.0 mEq/L. Urine studies show a pH of 6.0, sodium E. Plasmapheresis

461
VI-39. A 35-year-old woman presents with complaints of 37°C (98.6°F), heart rate of 105 beats/min, blood pressure
bilateral lower extremity edema, polyuria, and moderate of 145/95, respiratory rate of 21 breaths/min, and room
SECTION VI

left-sided flank pain that began approximately 2 weeks air oxygen saturation of 98%. His physical examination is
ago. There is no past medical history. She is taking no notable for left flank pain but no abdominal organomeg-
medications and denies tobacco, alcohol, or illicit drug aly or focal tenderness. An electrocardiogram shows sinus
use. Examination shows normal vital signs, including nor- tachycardia with nonspecific ST-T wave changes. Interna-
mal blood pressure. There is 2+ edema in bilateral lower tional normalized ratio is 2.0. His renal function is normal,
extremities. The 24-hour urine collection is significant and urine analysis shows many red blood cells, few white
for 3.5 g of protein. Urinalysis is bland except for the pro- blood cells, no bacteria, and no crystals. Which of the fol-
Disorders of the Kidney and Urinary Tract

teinuria. Serum creatinine is 0.7 mg/dL, and ultrasound lowing is the preferred diagnostic study?
examination shows the left kidney measuring 13 cm and
A. 24-Hour urine collection
the right kidney measuring 11.5 cm. You are concerned
B. Cystoscopy
about renal vein thrombosis. What test do you choose for
C. MRI
the evaluation?
D. Noncontrast CT scan
A. CT of the renal veins E. Ultrasound
B. Contrast venography
C. Magnetic resonance venography VI-43. In the patient described above, a noncontrast abdom-
D. 99Tc-labeled diethylene-triamine-pentaacetate acid inal CT is performed (Figure VI-43). Which of the follow-
(DTPA) imaging ing is the most likely diagnosis?
E. Ultrasound with Doppler evaluation of the renal
veins

VI-40. A 28-year-old woman who is in the 30th week of her


second pregnancy has been followed closely because of
mild hypertension. Her first pregnancy was complicated
by preeclampsia. She now complains of worsening fatigue
over the last day. Her blood pressure is 140/90, and she has
a heart rate of 84 beats/min and room air oxygen satura-
tion of 95%. Fetal monitoring shows no distress. Labora-
tory studies are notable for a hemoglobin of 6 g/dL (10 g/
dL 1 week ago) and platelet count of 80,000/dL (180,000/
dL 1 week ago). A peripheral blood smear shows schisto-
cytes. All of the following statements are true about her
condition EXCEPT:
A. Glucocorticoids are effective in reducing morbidity
and mortality.
B. Her liver function enzymes are likely elevated.
C. Preeclampsia predisposes to the condition. FIGURE VI-43 Used with permission from Dr. Stuart Silverman,
D. Renal failure is common. Brigham and Women’s Hospital.
E. The condition will likely resolve after delivery of the
fetus. A. Appendicitis
B. Nephrolithiasis
VI-41. Which of the following interventions has been shown C. Renal cell carcinoma
to decrease the recurrence of calcium oxalate renal stones? D. Pyelonephritis
A. A diet with abundant spinach and rhubarb E. Retroperitoneal hematoma
B. Aspirin
VI-44. Which of the following fluid or electrolyte abnormal-
C. Low-calcium diet (400 mg/day)
ities can be seen in some patients after the relief of bilateral
D. Thiazide diuretics
renal obstruction?
E. Vitamin C supplements
A. Hyperkalemia
VI-42. A 48-year-old man with diabetes mellitus, hyperlipi- B. Hypermagnesemia
demia, and atrial fibrillation presents to the emergency C. Hypernatremia
department for evaluation of left flank pain and groin pain D. Hypovolemia
that has been severe and present for approximately 3 hours. E. C and D
His medications include metformin, atorvastatin, and
warfarin. He is uncomfortable and has a temperature of

462
WWW.BOOKBAZ.IR
VI-45. A 54-year-old woman with a history of colon cancer A. CT of the abdomen/pelvis with oral contrast
treated with resection and chemotherapy 2 years earlier is B. Postvoid residual volume of bladder

SECTION VI
admitted to the hospital after routine lab work at her pri- C. Retrograde urography
mary care physician’s office showed a blood urea nitrogen D. Ultrasound of the abdomen/kidney
of 65 mg/dL and a creatinine of 4.5 mg/dL. She reports E. Urinary fractional excretion of sodium
mild fatigue and recent lower back pain, but otherwise
feels well. She does admit to recent nonsteroidal anti- VI-46. The pain associated with acute urinary tract obstruc-
inflammatory drug (NSAID) use but has not taken more tion is a result of which of the following?
than the recommended quantity. Aside from stopping the A. Compensatory natriuresis

ANSWERS
NSAID and avoidance of nephrotoxins, which of the fol- B. Decreased medullary blood flow
lowing studies should be the next step? C. Increased renal blood flow
D. Vasodilatory prostaglandins

ANSWERS

VI-1. The answer is D. (Chap. 303) The proximal tubule is responsible for reabsorbing ~60%
of filtered sodium chloride (NaCl) and water, as well as ~90% of filtered bicarbonate and
most critical nutrients such as glucose and amino acids. The proximal tubule uses both
cellular and paracellular transport mechanisms. The apical membrane of proximal tubu-
lar cells has an expanded surface area available for reabsorptive work created by a dense
array of microvilli called the brush border, and leaky tight junctions enable high-capacity
fluid reabsorption. Approximately 15–25% of filtered NaCl is reabsorbed in the loop of
Henle, mainly by the thick ascending limb. The distal convoluted tubule reabsorbs ~5%
of the filtered NaCl. This segment is composed of a tight epithelium with little water per-
meability. The collecting duct modulates the final composition of urine. The two major
divisions, the cortical collecting duct and inner medullary collecting duct, contribute to
reabsorbing ~4–5% of filtered Na+ and are important for hormonal regulation of salt and
water balance.

VI-2. The answer is B. (Chap. 304) Several antimicrobial agents are commonly associated with
acute kidney injury (AKI). Aminoglycosides (e.g., gentamicin) and amphotericin B both
cause tubular necrosis. Nonoliguric AKI (i.e., with a urine volume >400 mL/day) accom-
panies 10–30% of courses of aminoglycoside antibiotics, even when plasma levels are in
the therapeutic range. Aminoglycosides are freely filtered across the glomerulus and then
accumulate within the renal cortex, where concentrations can greatly exceed those of the
plasma. AKI typically manifests after 5–7 days of therapy and can present even after the
drug has been discontinued. Hypomagnesemia is a common finding. AKI secondary to
acute interstitial nephritis can occur as a consequence of exposure to many antibiotics,
including penicillins, cephalosporins, quinolones, sulfonamides, and rifampin. There is
no reason for obstructive nephropathy or ischemic injury by history. Renal ultrasound
shows no evidence of obstruction. There are no red or white cell casts or anything in the
history to suggest glomerulonephritis.

VI-3. The answer is C. (Chap. 304) A variety of electrolyte abnormalities can be seen in acute
kidney injury (AKI) including hyponatremia, hyperkalemia, metabolic acidosis, hyper-
phosphatemia, and hypocalcemia. Abnormalities in plasma electrolyte composition can
be mild or life-threatening. The dysfunctional kidney has limited ability to regulate elec-
trolyte balance. Administration of excessive hypotonic crystalloid or isotonic dextrose
solutions can result in hyposmolality and hyponatremia, which, if severe, can cause neu-
rologic abnormalities, including seizures. Marked hyperkalemia is particularly common

463

You might also like